MKSAP 17 Hospitalist Focused Content

Page 1

Medical Knowledge Self-Assessment ProgramÂŽ

PART A Cardiovascular Medicine Dermatology Gastroenterology and Hepatology Hematology and Oncology Neurology Rheumatology

Part A AMA PRA Category 1 Credits™ available until July 31, 2018.


Medical Knowledge Self-Assessment ProgramÂŽ

Cardiovascular Medicine

21 AMA PRA Category 1 Credits™ available until July 31, 2018.


Diagnostic Testing in Cardiology

TAbLE 2.

Risk Level at Which CHD Events Prevented Exceed GI Harms in Patients Taking Daily Aspirin for Primary Prevention Men 10-Year CHD

Women Riska

10-Year Stroke Riskb

Age 45-59 years

≥4%

Age 55-59 years

≥3%

Age 60-69 years

≥9%

Age 60-69 years

≥8%

Age 70-79 years

≥12%

Age 70-79 years

≥11%

CHD = coronary heart disease; GI = gastrointestinal. aRisk factors for CHD include age, diabetes mellitus, total cholesterol level, HDL cholesterol level, blood pressure, and smoking. CHD risk estimation tool: http://cvdrisk.nhlbi.nih. gov/calculator.asp. (Note: This is the Framingham risk score.) b

Risk factors for ischemic stroke include age, high blood pressure, diabetes mellitus, smoking, history of cardiovascular disease, atrial fibrillation, and left ventricular hypertrophy.

NOTE: This table applies to adults who are not taking NSAIDs and who do not have upper GI pain or a history of GI ulcers. Adapted with permission from U.S. Preventive Services Task Force. Aspirin for the prevention of cardiovascular disease: U.S. Preventive Services Task Force recommendation statement. Ann Intern Med. 2009 Mar 17;150(6):396-404. [PMID: 19293072]

Diagnostic Testing in Cardiology Clinical History and Physical Examination The initial step in evaluating for heart disease is a thorough history and physical examination. Specifically, a careful exploration of changes in functional status, associated symptoms, and the timing and nature of symptoms will help focus the assessment and guide selection of appropriate testing, if indicated. Cardiovascular testing provides both diagnostic and prognostic information and its use should be guided by symptoms, the level of risk for heart disease, and whether outcomes may be altered by interventions based on testing results.

Diagnostic Testing for Atherosclerotic Coronary Disease Cardiac Stress Testing Patients are referred for stress testing to establish the diagnosis of coronary artery disease (CAD) most often because of new onset of or a change in symptoms. The utility of stress testing should be interpreted in the context of the pretest likelihood of disease. Those with low probability of disease, such as younger patients, have a higher incidence of falsepositive tests and may undergo unnecessary testing without changing patient outcomes. Those with a high probability of disease should proceed directly to an invasive diagnostic strategy, such as cardiac catheterization, because the risk of a false-negative result and missed diagnosis is too high. Furthermore, a negative test in a high-risk patient would not significantly change the post-test probability of CAD, and therefore would not change management. Stress testing is most clinically appropriate in patients with an intermediate risk of CAD. It is these patients who, by the result of their

stress test, can be reclassified into higher or lower risk categories. Stress testing also has important prognostic value for predicting the risk of myocardial infarction and death in selected patients. For example, in patients with a previous history of CAD and worsening cardiac symptoms, stress testing is helpful to assess for possible recurrent or progressive disease. However, although the leading cause of death in patients with diabetes mellitus is cardiovascular disease, routine stress testing in asymptomatic patients with diabetes has not been shown to reduce mortality. The modalities able to detect cardiac ischemia are highly dependent on the degree of impairment of coronary blood flow. The earliest changes with mild stenosis are perfusion changes detectable only with highly sensitive modalities, such as nuclear or cardiac magnetic resonance (CMR) imaging. With progressive coronary occlusion, diastolic dysfunction followed by systolic dysfunction may be seen by imaging studies such as echocardiography. Only when there is significant coronary stenosis will electrocardiographic (ECG) changes be seen and symptoms occur. The many different types of tests to diagnose CAD can be broadly categorized as assessing either functional or anatomic evidence of ischemia. Functional studies evaluate for obstructive CAD from evidence of ECG changes, myocardial perfusion abnormalities, or wall motion abnormalities, usually under stress conditions. Anatomic studies assess percentage stenosis of the coronary vessels at rest, which can be visualized by single-photon emission CT (SPECT), PET/CT scan, or CMR imaging study. These imaging modalities may also be used to quantify infarction size and assess myocardial viability. More specific signs of ischemia such as reduced regional contractility can be assessed by echocardiography or MRI. Testing modalities for suspected CAD are summarized in Table 3. In intermediate-risk patients who are able to exercise and have a normal baseline ECG, the initial type of stress testing should be exercise stress testing. The additional prognostic information available with exercise, including functional 5


Diagnostic Testing in Cardiology

TAbLE 3.

Diagnostic Testing for Coronary Artery Disease

Diagnostic Test

Utility

Advantages

Limitations

Exercise ECG

Initial diagnostic test in most patients with suspected CAD

Data acquired on exercise capacity, blood pressure and heart rate response, and provoked symptoms

Not useful when baseline ECG is abnormal (LVH, LBBB, paced rhythm, WPW syndrome, >1 mm ST-segment depression)

Stress echocardiography

Recommended when baseline ECG is abnormal or when information on a particular area of myocardium at risk is needed

Exercise data acquired along with imaging for wall motion abnormalities to indicate ischemia

Image quality is suboptimal in some patients but can be improved with microbubble transpulmonary contrast

Allows evaluation of valve function and pulmonary pressures

Image interpretation is difficult when baseline wall motion abnormalities are present

Relatively portable and less costly than nuclear protocols

Diagnostic accuracy decreases with single-vessel disease or delayed stress image acquisition

Exercise Stress Testing

Entire study is completed in <1 h Nuclear SPECT perfusion

Recommended when baseline ECG is abnormal or when information on a particular area of myocardium at risk is needed With LBBB, conduction delay in the septum may cause falsepositive abnormality; this can be improved with the use of vasodilator stress

Gating (image acquisition coordinated with the cardiac cycle), use of higher energy agents such as technetium, and techniques used to correct for attenuation provide improved specificity

Attenuation artifacts can be caused by breast tissue or diaphragm interference Radiation exposure

Late reperfusion imaging allows evaluation of myocardial viability if thallium is used

Pharmacologic Stress Testing Dobutamine echocardiography

Recommended in patients who cannot exercise Recommended when information on an area of myocardium at risk is needed

Vasodilator nuclear perfusion (adenosine, dipyridamole, regadenoson)

Recommended in patients who cannot exercise May minimize septal abnormalities frequently seen with nuclear perfusion scanning in patients with LBBB

Because the patient is supine, images are acquired continuously, allowing the test to be stopped as soon as ischemia is evident

Dobutamine contraindications are severe baseline hypertension, unstable angina, and arrhythmias

Vasodilator stress testing may minimize effect of β-blockade on perfusion defect size

Contraindications are bronchospastic airway disease, theophylline use, sick sinus syndrome, and high-degree AV block

Can image sooner after myocardial infarction with vasodilator stress

β-Blockers must be withheld before the test

Caffeine must be withheld 24 hours before the test Adenosine or dipyridamole may cause chest pain, dyspnea, or flushing Radiation exposure

Dobutamine nuclear perfusion

Recommended in patients who cannot exercise and have contraindications to vasodilators Recommended when information on an area of myocardium at risk is needed

PET/CT

Provides best perfusion images in larger patients Provides data on myocardial perfusion, function, and viability

Has comparable sensitivity and specificity to exercise or vasodilator perfusion imaging for diagnosis of myocardial ischemia

Dobutamine contraindications are severe baseline hypertension, unstable angina, and arrhythmias β-Blockers should be withheld before the test Radiation exposure

Study duration is shorter and radiation dose is lower than conventional nuclear perfusion imaging

Not widely available

Absolute myocardial blood flow can be measured

Used with pharmacologic stress only (no exercise protocol)

Can be combined with CAC scoring

Radiation exposure

More expensive than other imaging modalities

(Continued on the next page)

6


Diagnostic Testing in Cardiology

TAbLE 3.

Diagnostic Testing for Coronary Artery Disease (Continued)

Diagnostic Test

Utility

Advantages

Limitations

Provides anatomic diagnosis of the presence and severity of CAD

Percutaneous revascularization can be performed following diagnostic study

Invasive

CAC testing

CAC testing is reasonable in asymptomatic patients at intermediate risk for CAD

CAC scores are predictive of cardiovascular risk in selected patients

Does not provide data on coronary luminal narrowing

Coronary CT angiography

Identifies anomalous coronary arteries

Coronary artery vessel lumen and atherosclerotic lesions can be visualized in detail

Requires high-resolution (64-slice) CT instruments

Other Tests Coronary angiography

Risks of vascular access and radiocontrast exposure (kidney dysfunction, allergy, bleeding) Radiation exposure

Useful for selected patients with intermediate risk for CAD

Radiation exposure

Does not provide detailed images of distal vessel anatomy Catheterization will be needed if intervention is planned Ability to quantify lesion severity can be limited by significant calcification Radiation and radiocontrast exposure

CMR imaging

Gadolinium-enhanced images identify viable and infarcted myocardium Identifies anomalous coronary arteries

Accurate test for myocardial viability

Some patients experience claustrophobia May be contraindicated in patients with pacemaker, ICD, or other implanted device Gadolinium is contraindicated in kidney failure Sinus rhythm and a slower heart rate are needed for improved image quality Limited availability and expertise

AV = atrioventricular; CAC = coronary artery calcium; CAD = coronary artery disease; CMR = cardiac magnetic resonance; ECG = electrocardiography; ICD = implantable cardioverter-defibrillator; LBBB = left bundle branch block; LVH = left ventricular hypertrophy; SPECT = single-photon emission CT; WPW = Wolff-Parkinson-White.

CONT.

capacity and heart rate and blood pressure response, can be utilized in prediction models such as the Duke treadmill score, which factors development of symptoms, degree of ST-segment depression, and exercise duration to provide incremental prognostic information for 5-year mortality risk. Heart rate recovery is another powerful predictor; patients with a heart rate drop of less than 12/min in the first minute after cessation of exercise have a higher mortality rate. Ischemia is identified on the basis of the development of 1 mm or greater of horizontal or downsloping ST depression with exercise (Figure 1), but the coronary territory involved cannot be localized based on the ECG changes alone. Ideally, patients should exercise for 6 to 12 minutes to provide adequate time for development of maximal metabolic demand. Although achieving 85% of the age-predicted maximal heart rate (PMHR) is considered adequate for diagnosis of ischemia, as heart rate and blood pressure are the major determinants

of myocardial oxygen demand, patients should continue to exercise until limited by symptoms. Achieving a rate pressure product (heart rate Ă— systolic blood pressure) of at least 25,000 is also considered an adequate workload, as this measure reflects left ventricular myocardial performance. A standard Bruce protocol increases the speed and grade of the treadmill every 3 minutes, and patients who have poor functional capacity and cannot achieve at least the first stage of the Bruce protocol (5 metabolic equivalents [METs]) have significantly higher all-cause mortality. Stress tests should be terminated when the patient has exerted maximal effort and achieved at least 85% PMHR, the patient requests to stop or experiences significant anginal or other physical symptoms, or when other adverse markers develop, such as exertional hypotension, significant hypertension, ST-segment elevation or significant ST-segment depression, or ventricular or supraventricular arrhythmias. 7


Self-Assessment Test

Self-Assessment Test

Which of the following is the most appropriate next step in treatment? (A) (B) (C) (D)

Discontinue hydrochlorothiazide Dual-chamber pacemaker Initiate lisinopril Surgical myectomy

Item 6 A 71-year-old man is evaluated in the emergency department for severe pain in the chest and back that was abrupt in onset and has persisted for 3 hours. He has no abdominal pain, leg pain, or neurologic symptoms. His medical history is notable for hypertension. Medications are amlodipine and lisinopril. On physical examination, the patient is afebrile, blood pressure is 180/100 mm Hg in both arms, pulse rate is 98/min, and respiration rate is 18/min. Oxygen saturation is 96% on ambient air. Cardiac auscultation discloses an S4 gallop but no murmur. Pulmonary examination is normal. Pulses are symmetric and equal in all extremities. Laboratory studies show a D-dimer level of 1.2 Âľg/mL (1.2 mg/L) and a serum creatinine level of 1.0 mg/dL (88.4 Âľmol/L). Initial serum cardiac troponin I level is not elevated. Electrocardiogram shows left ventricular hypertrophy with repolarization abnormalities. Chest radiograph shows an enlarged cardiac silhouette. Chest CT scan with intravenous contrast demonstrates a focal penetrating ulcer in the thoracic descending aorta (shown).

Item 7 A 52-year-old woman is evaluated for fatigue and lower extremity swelling. One year ago, she had acute idiopathic pericarditis treated with anti-inflammatory medications and colchicine. Symptoms initially improved and her medications were discontinued. However, for the past 3 months, she has had worsening symptoms of exertional fatigue and edema in both legs. She currently takes no medications. On physical examination, she is afebrile, blood pressure is 130/78 mm Hg, pulse rate is 88/min, and respiration rate is 16/min. Pulsus paradoxus of 15 mm Hg is present. The estimated central venous pressure is 10 cm H2O, and the jugular venous pulse contour shows diminished y descents. The lungs are clear to auscultation. Heart sounds are normal, with no rubs or gallops. Hepatomegaly is present, and peripheral edema is noted in both lower extremities up to the knees. A 12-lead electrocardiogram is normal. Echocardiogram shows the cardiac chambers to be normal in size and function with a moderate circumferential pericardial effusion. A CT of the heart shows normal pericardial thickness. A pericardiocentesis fails to resolve the elevated right atrial pressure documented on right heart catheterization. Which of the following is the most likely diagnosis? (A) (B) (C) (D)

Cor pulmonale Effusive constrictive pericarditis Heart failure Recurrent acute pericarditis

Item 8 A 58-year-old man is evaluated during a routine appointment. He is asymptomatic. He was diagnosed with type 2 diabetes mellitus 4 years ago and has hypertension, dyslipidemia, and obesity. His medications are enteric-coated low-dose aspirin, lisinopril, fluvastatin (20 mg/d), and metformin. His calculated 10-year risk of atherosclerotic cardiovascular disease (ASCVD) using the Pooled Cohort Equations is 10%. On physical examination, blood pressure is 126/78 mm Hg and pulse rate is 72/min. The remainder of the examination is normal. Laboratory studies: Total cholesterol LDL cholesterol HDL cholesterol Triglycerides

186 mg/dL (4.82 mmol/L) 123 mg/dL (3.19 mmol/L) 44 mg/dL (1.14 mmol/L) 109 mg/dL (1.23 mmol/L)

Which of the following is the most appropriate immediate next step in management?

Which of the following is the most appropriate statin management?

(A) Heparin followed by warfarin (B) Endovascular stenting (C) Intravenous β-blockade followed by intravenous sodium nitroprusside (D) Open surgical repair

(A) (B) (C) (D) (E)

122

Increase fluvastatin to 40 mg/d Switch to atorvastatin, 40 mg/d Switch to lovastatin, 20 mg/d Switch to pravastatin, 20 mg/d Switch to simvastatin, 10 mg/d


use or squat-to-stand maneuvers) and afterload (for example, from expiration or vasodilator therapy) and by increases in myocardial contractility (for example, from digoxin therapy). For patients with symptoms of HCM, the initial treatment is medical therapy that addresses the factors that predispose toward LVOT obstruction. Use of hydrochlorothiazide and other diuretics, particularly in high doses, will exacerbate the propensity towards dynamic LVOT obstruction and, therefore, should be avoided in patients with HCM. Similarly, vasodilator therapy also should not be used. Negative inotropic agents, such as β-receptor antagonists, calcium channel blockers, and disopyramide, are the cornerstone of medical therapy in these patients. This patient was previously stable on hydrochlorothiazide; however, HCM and its associated pathophysiology can manifest at any age and result in symptoms of heart failure. Dual-chamber pacemaker implantation previously was considered to be a therapeutic option but was found to be relatively ineffective in randomized clinical trials of patients with HCM. These trials showed a significant placebo effect (40%50%) from dual-chamber pacing without clear clinical benefit. ACE inhibitors, such as lisinopril, reduce afterload and can exacerbate LVOT obstruction, and therefore they should be avoided in patients with HCM. For patients with drug-refractory, severely symptomatic obstructive HCM, septal reduction therapy with surgical myectomy or alcohol septal ablation may be considered. Neither myectomy nor alcohol ablation is recommended for patients with the minimal symptoms that this patient has or for those without an adequate trial of appropriate pharmacotherapy. Key PoinT

• Diuretics, particularly in high doses, will exacerbate the propensity towards dynamic left ventricular outflow tract obstruction and, therefore, should be avoided in patients with hypertrophic cardiomyopathy.

Bibliography Gersh BJ, Maron, BJ, Bonow, RO, et al; American College of Cardiology Foundation/American Heart Association Task Force on Practice Guidelines. 2011 ACCF/AHA Guideline for the Diagnosis and Treatment of Hypertrophic Cardiomyopathy: a report of the American College of Cardiology Foundation/ American Heart Association Task Force on Practice Guidelines. Developed in collaboration with the American Association for Thoracic Surgery, American Society of Echocardiography, American Society of Nuclear Cardiology, Heart Failure Society of America, Heart Rhythm Society, Society for Cardiovascular Angiography and Interventions, and Society of Thoracic Surgeons. J Am Coll Cardiol. 2011 Dec 13;58(25):e212-60. [PMID: 22075469]

Item 6

Answer:

C

Educational Objective: Treat a penetrating atherosclerotic ulcer in the descending aorta. The most appropriate management for this patient is intravenous β-blockade followed by intravenous sodium nitroprusside. The CT scan (shown, see top of next column) reveals a focal penetrating atherosclerotic ulcer (PAU) in the proximal descending aorta (arrow), a type B acute aortic syndrome. PAU is a focal defect or lesion occurring at the site of an intimal atherosclerotic plaque. Patients tend to be older and with greater cardiovascular comorbidity. PAU occurs most

commonly in the descending aorta, which may reflect a greater burden of atheromatous disease at this site. Elevation of the D-dimer level frequently accompanies acute aortic syndromes. Uncomplicated type B acute aortic injury is best treated medically, initially with β-blockade to decrease the heart rate to below 60/min followed by a parenteral arterial vasodilator as needed to control blood pressure. Chest pain is a common presenting symptom in the emergency department, and three potentially lethal causes must be rapidly excluded: acute myocardial infarction, pulmonary embolism, and an acute aortic syndrome. Anticoagulation therapy would be considered standard of care if an acute coronary syndrome were suspected. In this patient, although a non–ST-elevation myocardial infarction cannot be excluded based on the electrocardiogram, it is less likely given his normal troponin I level; the troponin level would likely be elevated after 3 hours of ongoing cardiac ischemia. The patient also has a low pretest probability of pulmonary embolism by Wells scoring. Given his clinical and radiographic presentation, neither alternative diagnosis to an acute aortic syndrome is likely, and anticoagulation is not indicated in acute aortic syndrome. Malperfusion syndromes are uncommon with an aortic penetrating atherosclerotic ulcer but could occur with transformation to classic dissection. In such cases, emergency endovascular stenting with or without fenestration may be necessary. This patient shows no evidence of lower limb or visceral malperfusion, and catheterization with endovascular repair is not indicated. Emergency open surgery is rarely indicated in this setting with the advent of endovascular therapies. Key PoinT

• Uncomplicated type B acute aortic injury is best treated medically, initially with β-blockade followed by a parenteral arterial vasodilator to control blood pressure.

Bibliography Nienaber CA, Powell JT. Management of acute aortic syndromes. Eur Heart J. 2012 Jan;33(1):26-35b. [PMID: 21810861]

159

Answers and Critiques

Answers and Critiques


Medical Knowledge Self-Assessment ProgramÂŽ

Dermatology

12 AMA PRA Category 1 Credits™ available until July 31, 2018.


Dermatologic Urgencies and Emergencies

TAblE 28.

Causes of Retiform Purpura and Associated Findings

Etiology

Associated Findings and Causes

Angioinvasive fungal infection

Infection caused by Mucorales, Aspergillus, Fusarium, Pseudallescheria Neutropenic, solid-organ transplant recipients, burn victims Infects locally but can become systemic

Necrotizing fasciitis

Pain out of proportion to examination History of preceding trauma Can be a mixed infection or due to Streptococcus Surgical emergency; imaging can be supportive but should not delay surgical exploration

Immune-mediated vasculitis

Polyarteritis nodosa ANCA-associated vasculitides: Eosinophilic granulomatosis with polyangiitis Granulomatosis with polyangiitis Microscopic polyangiitis Connective tissue disease-associated (systemic lupus erythematosus, rheumatoid arthritis, dermatomyositis) Evaluate for ANCA, signs of connective tissue disease, other end-organ damage (kidney, lung, eye, gastrointestinal, musculoskeletal)

Calciphylaxis

Associated with advanced kidney disease Extremely painful Extensive forms occur on central, adipose sites (breast, abdomen, hips) Limited forms often occur distally

Thromboembolism

DIC, purpura fulminans Systemic inflammation due to infection or other insult Evidence of organ ischemia and uncontrolled bleeding from multiple sites

Drugs

Levamisole-adulterated cocaine: tender purpura on the ears is prominent Warfarin skin necrosis: rare, occurs in first week of therapy, commonly involves adipose areas such as the breasts, abdomen, and hips Heparin necrosis: rare, onset in the first 5 to 10 days of therapy, can occur at the sites of injection or elsewhere, may or may not be associated with thrombocytopenia

ANCA = antineutrophil cytoplasmic antibody; DIC = disseminated intravascular coagulation.

CONT.

The history can yield important clues to the cause such as infection, thrombotic disease, recent intravascular procedures, spontaneous abortion, solid organ or hematologic malignancy, or prescribed medication or illicit drug reactions. Laboratory tests, directed by the history and physical examination, are indicated to investigate the cause and assess for end-organ damage. In select patients, skin biopsy may be helpful in establishing the diagnosis. Large incisional biopsies, which include the dermis and subcutis, are preferred over punch biopsies. An additional specimen is sent for tissue culture if infection is suspected. Key Point

• Retiform purpura often indicates infection, thrombotic disease, recent intravascular procedures, spontaneous abortion, solid organ or hematologic malignancy, or prescribed medication or illicit drug reactions.

Erythema Multiforme, StevensJohnson Syndrome, and Toxic Epidermal Necrolysis Erythema multiforme (EM) can be recognized by the target lesions on the palms and soles as well as mucosal erosions, most frequently in the mouth (Figure 116). In contrast, typical target lesions are rare in Stevens-Johnson syndrome (SJS) and toxic epidermal necrolysis (TEN). SJS and TEN are distinguished by the amount of skin involved by blisters or erosions (Table 29). SJS is defined as affecting less than 10% body surface area (BSA), and TEN affects more than 30% BSA. When 10% to 30% BSA is involved, it is considered SJS-TEN overlap with a mortality rate between the two. SJS and TEN are rare, with a prevalence of 1:100,000 for SJS and 1:1,000,000 for TEN. The conditions on the SJS-TEN spectrum can cause significant pain and scarring of involved mucosal surfaces, although the mortality risk is associated with the more severe end of the 69


except in young children. The rash associated with scabies consists of pink papules and macules, some with scale or serous crust, as well as some linear macules that are the burrows of the mites. The patient likely became infested when he stayed with the friend with a rash. After infestation, the rash may take up to a month to become clinically obvious. Allergic contact dermatitis may involve the dorsal hands, finger web spaces, and skin on the volar wrists, particularly when the palms are spared, but more commonly presents with erythema, edema, and papulovesicles, which are not present in this patient. Lesions are frequently geometric and have sharp cut-offs and a linear component to the morphology. There is typically a delayed onset of 1 to 3 days after exposure. Bedbugs (genus Cimex) are another parasite of humans, but while the scabies mite lives on the human host, the bedbug lives off the human body. The bedbug is active during the night and typically bites areas of skin that are not covered by nightclothes. The face (including the eyelid), neck, hands, arms, and waist may have evidence of bites in the morning or may be delayed by 7 to 10 days. The bites are itchy and may be small pink-red papules or urticarial plaques 1 to 2 cm in diameter. Bedbugs live in crevices of mattresses, furniture cushions, suitcases, and walls. Tinea manuum is a dermatophyte infection involving the hands and feet. Tinea manuum characteristically involves only one hand (and two feet). The findings are characterized by a dry scale, and if chronic, nails can be involved. The diagnosis of tinea manuum is unlikely if the feet are not involved. Key PoinT

• The rash associated with scabies consists of pink papules and macules, some with scale or serous crust as well as some linear macules that are the burrows of the mites.

Bibliography Heukelbach J, Feldmeier H. Scabies. Lancet. 2006 May 27;367(9524):1767-74. [PMID: 16731272]

Item 24

Answer:

A

Educational Objective: Diagnose allergic contact dermatitis. A pruritic geometric patch typical of allergic contact dermatitis, and in this patient, the clonidine patch is the likely cause. Contact dermatitis is an example of an “outside job” when a rash is in linear or geometric patterns because the rash corresponds exactly to where the allergen touches the skin from the outside. Allergic contact dermatitis is a hypersensitivity reaction to a specific chemical. With repeated exposure to the chemical, a pruritic eczematous dermatitis develops on the area that was exposed. In exuberant cases, pinpoint fleshcolored to red papules develop in the vicinity of exposure or diffusely over the body. The clonidine patch has been associated with allergic contact dermatitis. Although many

transdermal drug delivery systems may cause allergic contact dermatitis, the clonidine patch tends to have a higher incidence, possibly owing to its formulation that allows the patch to be left in place for up to 7 days. The patient’s adjacent areas of hyperpigmentation likely represent resolving inflammation from prior applications of the patch. A fixed drug eruption is typically a purple patch that is painful and is not localized to the area underneath a patch. Fixed drug eruptions occur in the same location (fixed) each time a patient is exposed to the same medication. Lips, genitals, and hands are commonly involved. Common drug culprits include over-the-counter medications such as pseudoephedrine, NSAIDs, sulfonamide medications, and other antibiotics. Irritant contact dermatitis is a direct toxic effect on the epidermis from exposure to a chemical such as a cleaning agent, other caustic substance, or repeated wetting and drying and is not mediated by the immune system. Irritant contact dermatitis would present with a generalized dermatitis in all of the exposed areas but not in a geometric pattern. Tinea corporis is a dermatophyte infection that may occur in healthy people, although patients with compromised immune systems (such as with diabetes mellitus) may be more susceptible. The classic findings are a pruritic, circular or oval, erythematous, scaling patch or plaque that spreads centrifugally with central clearing. This patient’s skin findings are not consistent with this diagnosis. Key PoinT

• Allergic contact dermatitis is a hypersensitivity reaction to a specific chemical, and with repeated contact with the chemical, a geometric pruritic eczematous dermatitis develops on the area that was exposed.

Bibliography Corazza M, Mantovani L, Virgili A, Strumia R. Allergic contact dermatitis from a clonidine transdermal delivery system. Contact Dermatitis. 1995 Apr;32(4):246. [PMID: 7600788]

Item 25

Answer:

D

Educational Objective: Treat toxic epidermal necrolysis. This patient has toxic epidermal necrolysis (TEN), and he should be admitted to the hospital and any medications that are possible triggers (minocycline) should be stopped immediately. The most commonly implicated medications are antiepileptic agents, especially carbamazepine, lamotrigine, and phenytoin. Sulfonamides, fluoroquinolones, β-lactam antibiotics, minocycline, pantoprazole, sertraline, NSAIDs (oxicam and acetic acid type), tramadol, and allopurinol are also frequent causes. The treatment of TEN begins with drug cessation and aggressive ICU or burn center care. Stevens-Johnson syndrome (SJS) and TEN are related clinical syndromes that are characterized by acute epidermal necrosis. The classification of SJS and TEN is determined by the percentage of body surface area with epidermal detachment: SJS involves less than 10%, SJS-TEN overlap involves 131

Answers and Critiques

Answers and Critiques


Answers and Critiques

CONT.

Answers and Critiques

10% to 30%, and TEN involves greater than 30%. TEN is a rare disease, with a prevalence of 1:1,000,000. TEN is almost exclusively caused by medications, whereas erythema multiforme and SJS can also be triggered by vaccines or infection. SJS and TEN occur within 8 weeks of drug initiation, often between 4 and 28 days. Patients may have flu-like symptoms for 1 to 3 days prior to the skin eruption. Initially, red-purple macules or patches develop on the trunk and extremities, which enlarge and coalesce. Skin pain is prominent, in contrast to the pruritus associated with a common drug exanthema. Vesicles, bullae, and erosions reflect the epidermal necrosis seen on biopsy. Nikolsky sign (the shearing off of the epidermis with lateral pressure on the skin) is present. Two or more mucosal surfaces, such as the eyes, nasopharynx, mouth, and genitals, are involved in more than 80% of patients. Systemic inflammation can result in pneumonia, hepatitis, nephritis, arthralgia, and myocarditis. Infliximab is a monoclonal antibody that is used for several conditions including psoriasis; however, it has no role in the management of TEN. There is evidence to show that treatment with systemic glucocorticoids, such as intravenous methylprednisolone, can increase mortality rates in patients with TEN. Infection is a risk for patients with SJS or TEN because of the extensive damage to the skin barrier; however, antibiotics, such as vancomycin and ceftriaxone, are not started without signs or symptoms of infection. Key PoinT

• The treatment of toxic epidermal necrolysis begins with discontinuing the offending drug and providing aggressive supportive care, such as that received in an ICU or burn center.

Bibliography Bastuji-Garin S, Fouchard N, Bertocchi M, et al. SCORTEN: a severity-ofillness score for toxic epidermal necrolysis. J Invest Dermatol. 2000; 115:149–53. [PMID: 16679892]

Item 26

Answer:

D

Educational Objective: Diagnose subacute cutaneous lupus erythematosus. This patient has a typical eruption of subacute cutaneous lupus erythematosus (SCLE). Although associated with systemic lupus erythematosus (SLE), it is a distinct disorder that may present with different cutaneous findings than those usually associated with SLE. There are two presentations of SCLE: annular scaly patches on the upper back and sunexposed areas or a more psoriasiform eruption that can have less distinctive morphology but also occurs in sun-exposed areas. SCLE is associated with a number of medications believed to trigger the disorder, including hydrochlorothiazide, calcium channel blockers, ACE inhibitors, terbinafine, and the tumor necrosis factor α (TNF-α) inhibitors. This patient is taking hydrochlorothiazide, which was the first drug reported in association with SCLE and remains a common cause of drug-induced SCLE. Patients with SCLE tend to be very 132

photosensitive, may have antinuclear antibody positivity, and are often positive for anti-Ro/SSA antibodies (and may be positive for anti-La/SSB antibodies). Patients with drug-induced SCLE may be antihistone antibody positive (about one third to one half of patients), but antihistone antibody positivity is not necessary to make the diagnosis. Compared patients with SLE, those with SCLE tend to have fewer systemic findings, although a subset of patients may have kidney involvement. Notably, all women with anti-ro/SSA and anti-La/SSB antibodies are at risk for having newborns with neonatal lupus, and pregnant patients or mothers of newborns with this pattern of skin eruption or serologic antibody profiles should be counseled and their babies screened accordingly. For druginduced SCLE, discontinuing the inciting medication is essential to treatment. Dermatomyositis may present without muscle inflammation, but this patient’s skin eruption is characteristic of SCLE. The rash of dermatomyositis is accentuated around the eyes (heliotrope rash), around the lateral shoulders (“shawl sign”), and involves the knuckles of the metacarpal and interphalangeal joints (Gottron sign). Pemphigus foliaceus is an autoimmune blistering disease in which the autoantigen is in the upper regions of the epidermis, leading to superficial blisters that rapidly rupture and cause dried crusts over inflamed skin, which have been described as looking like “cornflakes.” The morphology and distribution of the eruption described in this patient would be atypical, and patients with pemphigus foliaceus are not often positive for anti-Ro/SSA antibodies or high-titer antinuclear antibodies. Psoriasis generally occurs over joints or at areas of trauma or friction, is not generally annular, and should not be photodistributed. Classic plaque psoriasis is characterized by pink plaques with silvery scale over the knees, elbows, and often around the umbilicus and gluteal cleft. Key PoinT

• Patients with subacute cutaneous lupus erythematosus have annular scaly patches that are typically drug or sunlight induced.

Bibliography Lowe G, Henderson CL, Grau RH, Hansen CB, Sontheimer RD. A systematic review of drug-induced subacute cutaneous lupus erythematosus. Br J Dermatol. 2011 Mar;164(3):465-72. [PMID: 21039412]

Item 27

Answer:

D

Educational Objective: Treat a hospital-acquired skin infection. This patient should be treated with vancomycin. Hospitalacquired skin and soft-tissue infections should be treated as a methicillin-resistant Staphylococcus aureus (MRSA) infection until culture results are received and therapy can be tailored appropriately. Hospital-acquired skin infections are increasingly caused by MRSA, and coverage against this organism in the hospital setting is important to prevent


Medical Knowledge Self-Assessment ProgramÂŽ

Gastroenterology and Hepatology

16 AMA PRA Category 1 Credits™ available until July 31, 2018.


Gastroenterology and Hepatology Disorders of the Esophagus Symptoms of Esophageal Disorders Dysphagia Dysphagia is the awareness of food not passing during the swallowing process. The swallowing mechanism is made up of two distinct components: 1. The oropharyngeal process is the passage of the food bolus from the mouth into the hypopharynx and upper esophagus. 2. The esophageal process is the passage of the food bolus through the upper esophagus into the stomach. Patients typically describe a feeling of food “getting stuck” or being blocked while eating. Identifying which component of the swallowing process (oropharyngeal or esophageal) is abnormal is key for formulating the differential diagnosis and devising a diagnostic and management plan. There are many causes of dysphagia, which are described in Table 1.

Oropharyngeal Dysphagia Patients with oropharyngeal dysphagia (also known as transfer dysphagia) are unable to initiate the swallowing process despite several attempts to swallow. Patients typically describe coughing, choking, and nasal regurgitation. Choking occurs owing to failure to clear food from the epiglottis and may lead to aspiration. Oropharyngeal dysphagia often occurs within 1 second of starting the swallowing process. Recurrent episodes of pulmonary infections (including pneumonia) can be a result of chronic aspiration. Symptoms of underlying systemic neurologic disease include hoarseness (laryngeal nerve dysfunction) and dysarthria (weakness of the soft palate or pharyngeal constrictors). Regurgitation of undigested food hours after eating or reports of a gurgling noise in the neck are symptoms of Zenker diverticulum. The initial test of choice for evaluation of oropharyngeal dysphagia is a modified barium swallow, also known as videofluoroscopy. The test begins with a liquid phase, which is followed by a solid phase if the liquid phase is not diagnostic. If results of the modified barium swallow are normal, oropharyngeal dysphagia is excluded and further evaluation should focus on the possibility of esophageal dysphagia. Oropharyngeal forms of dysphagia are often managed with dietary adjustment and incorporation of swallowing exercises with the assistance of a speech pathologist.

Key Point

• The initial test of choice for evaluation of oropharyngeal dysphagia is a modified barium swallow, also known as videofluoroscopy.

Esophageal Dysphagia Patients with esophageal dysphagia often localize discomfort to the lower sternum and do not report problems initiating the swallowing process. Dysphagia to solid foods suggests mechanical obstruction; dysphagia to liquids or both solids and liquids suggests a motility disorder. Patients with achalasia describe regurgitation of nonacidic undigested food in combination with dysphagia to solids and liquids. Esophageal spasm is associated with chest pain that may be triggered by consuming liquids of extreme hot or cold temperatures. Systemic symptoms such as Raynaud phenomenon in the setting of heartburn suggest systemic sclerosis, a cause of both mechanical obstruction and motility disorders. Luminal causes of obstruction may include benign strictures, malignancy, esophageal webs, or a Schatzki ring (Figure 1). Upper endoscopy is the most appropriate test for esophageal dysphagia; it allows for both diagnostic intervention (biopsies and inspection) and therapeutic intervention (dilation). Treatment is guided by the underlying pathology causing the dysphagia, as discussed in later sections. Key Points

• Esophageal dysphagia to solid foods suggests mechanical obstruction; dysphagia to liquids or both solids and liquids suggests a motility disorder. • Upper endoscopy is the most appropriate test for esophageal dysphagia; it allows for both diagnostic intervention (biopsies and inspection) and therapeutic intervention (dilation).

Reflux and Chest Pain Reflux is the regurgitation of acidic contents from the stomach into the esophagus or back of the throat. Untreated reflux can lead to development of a peptic stricture, causing solid-food dysphagia. Pyrosis, also known as heartburn, is the most common digestive complaint in Western populations. Heartburn often occurs approximately 1 hour after eating and is temporarily relieved with antacids. Factors that can trigger or exacerbate reflux symptoms are listed in Table 2. Patients may have esophageal chest pain symptoms that are identical to those of cardiac chest pain. Esophageal chest 1


Disorders of the Esophagus

TablE 1.

Causes of Dysphagia

Condition

Diagnostic Clues Oropharyngeal Dysphagia

Structural disorders Cervical osteophytes

High dysphagia, degenerative joint disease

Cricoid webs

High dysphagia, iron deficiency

Pharyngoesophageal (Zenker) diverticulum

Aspiration, neck mass, and regurgitation of foul-smelling food

Thyromegaly

Neck mass

Neurologic/myogenic disorders Amyotrophic lateral sclerosis

Upper and lower motoneuron signs, fasciculations

Central nervous system tumor

Headache, vision changes, nausea, seizures, balance problem

Stroke

Focal neurologic deficits

Muscular dystrophy

Slow progression of muscular weakness over years

Myasthenia gravis

Weakness with repetitive activity

Multiple sclerosis

Optic neuritis

Parkinson disease

Bradykinesia, rigidity, tremor

Dementia

Altered cognition

Sjรถgren syndrome

Dry mouth, dry eyes Esophageal Dysphagia

Structural disorders Dysphagia lusoria (vascular dysphagia)

Vascular extrinsic compression on the esophagus on imaging

Epiphrenic/traction diverticulum

Outpouching of the esophagus at any level on imaging

Esophageal strictures

Intermittent dysphagia, especially for solid food; history of reflux

Eosinophilic esophagitis

Food impactions, atopic history, rings or strictures on endoscopy

Esophageal webs or rings

Usually incidental finding, may be associated with iron deficiency anemia

Neoplasms

Rapidly progressive dysphagia for solids, then liquids; anorexia; weight loss

Motility disorders Achalasia

Concomitant liquid and solid dysphagia

Diffuse esophageal spasm

Chest pain

Systemic sclerosis

Tight skin, telangiectasias, sclerodactyly, GERD, Raynaud phenomenon

GERD = gastroesophageal reflux disease.

CONT.

pain may be induced by physical activity, similarly to cardiac chest pain. The most common cause of noncardiac chest pain is untreated gastroesophageal reflux disease (GERD), followed by motility disorders. Cardiac causes must be ruled out before chest pain is attributed to an esophageal cause. A trial of an acid-reducing agent such as a proton pump inhibitor (PPI) may alleviate symptoms, confirming the diagnosis of GERD. If symptoms are unresponsive to a PPI trial, additional evaluation with upper endoscopy, ambulatory pH testing, and/or esophageal manometry may be warranted. For diagnosis and management of reflux, see Gastroesophageal Reflux Disease. Key Point

โ ข Cardiac causes must be ruled out before chest pain is attributed to an esophageal cause. 2

Odynophagia Odynophagia is defined as pain with swallowing and is an indication of inflammation in the esophagus. Odynophagia is a sign of mucosal injury of the esophagus leading to ulceration. The most common causes of odynophagia are caustic ingestion, pill-induced damage, and infection with Candida, herpesvirus, or cytomegalovirus. Rarely, it has been associated with severe GERD or esophageal cancer. Upper endoscopy is the diagnostic test of choice for visual inspection and obtaining tissue biopsies.

Globus Sensation Globus is the sensation of tightness or a lump in the throat. Globus occurs between swallows and is not related to meals. Stress, psychiatric disorders (anxiety, panic disorders,


Each of the numbered items is followed by lettered answers. Select the ONE lettered answer that is BEST in each case.

Item 1 A 55-year-old man is evaluated during a routine examination. He feels well other than mild knee pain. He drinks six to eight cans of beer per night. He has no personal history of liver disease, but his older brother was recently diagnosed with hereditary hemochromatosis. On physical examination, vital signs are normal; BMI is 24. He is tanned on sun-exposed body surfaces. Cardiac examination is normal. Abdominal examination reveals hepatomegaly. Bilateral bony hypertrophy of the knees is noted. Laboratory studies: Alanine aminotransferase Aspartate aminotransferase Ferritin Transferrin saturation

70 U/L 160 U/L 592 ng/mL (592 µg/L) 40%

Genetic testing for hemochromatosis reveals heterozygosity for C282Y. Abdominal ultrasound reveals a change in liver echotexture consistent with fatty changes. Which of the following is the most appropriate treatment? (A) (B) (C) (D)

Administer deferoxamine Perform phlebotomy Repeat serum iron tests now Stop alcohol intake

Item 2 A 40-year-old man is evaluated for a 6-month history of intermittent episodes of two to four loose stools per day. When he has diarrhea, he also notices crampy abdominal pain and bloating. He has not had nausea, vomiting, anorexia, fever, melena, hematochezia, recent travel, or any new medications, including antibiotics. He is overweight but has been exercising and watching his diet for the past 6 months, and he has intentionally lost 6.8 kg (15.0 lb). The main change in his diet has been switching to diet soda and using sugar-free sweeteners. He takes no medications. On physical examination, vital signs are normal. The abdomen is obese but soft with normal bowel sounds and no distention or tenderness. Which of the following is the most appropriate management? (A) (B) (C) (D) (E)

Abdominal CT scan Colonoscopy with biopsies Discontinuation of sugar-free sweeteners Gluten-free diet Tissue transglutaminase IgA antibody testing

Item 3 A 68-year-old man is evaluated for new-onset ascites with lower-extremity edema. Symptoms have increased gradually over the past 4 weeks. He has consumed three alcoholic beverages per day for many years. His medical history is

notable for coronary artery bypass graft surgery 8 months ago and dyslipidemia. His medications are low-dose aspirin, atorvastatin, and metoprolol. On physical examination, temperature is 36.8 °C (98.2 °F), blood pressure is 122/84 mm Hg, pulse rate is 64/min, and respiration rate is 16/min; BMI is 28. Cardiac examination reveals an elevated jugular venous pressure, a normal S1 and S2, and no murmurs. Pulmonary examination findings are normal. Abdominal examination reveals hepatomegaly, distention, dullness to percussion over the flanks, and a positive fluid wave. There is 2+ pitting edema of the lower extremities. Laboratory studies reveal a serum albumin level of 3.5 g/dL (35 g/L). Other studies, including serum alanine aminotransferase and aspartate aminotransferase levels, are normal. Paracentesis reveals a total nucleated cell count of 120/µL with 30% polymorphonucleocytes. Ascitic fluid albumin level is 2.3 g/dL (23 g/L) and total protein is 3.5 g/ dL (35 g/L). Which of the following is the most likely cause of this patient’s ascites? (A) (B) (C) (D)

Alcoholic cirrhosis Constrictive pericarditis Nonalcoholic cirrhosis Tuberculous peritonitis

Item 4 A 48-year-old woman is evaluated at an urgent care center for a 2-day history of severe retrosternal chest pain. She has not had heartburn or dysphagia. She is finishing a course of treatment for Lyme disease, but she has no other medical problems and otherwise feels well. Her only medication is doxycycline. Physical examination is unremarkable, and laboratory studies are normal. Which of the following is the most likely diagnosis? (A) (B) (C) (D)

Barrett esophagus Candida esophagitis Eosinophilic esophagitis Pill-induced esophagitis

Item 5 A 56-year-old woman is hospitalized for new-onset confusion. She has a history of decompensated hepatitis B cirrhosis with chronic ascites. Her medications are spironolactone, furosemide, lactulose, and entecavir. On physical examination, temperature is 36.8 °C (98.2 °F), blood pressure is 118/62 mm Hg, pulse rate is 88/ min, and respiration rate is 20/min; BMI is 23. She is disoriented to time and date. Asterixis is noted. The abdomen is moderately distended with ascites. Laboratory studies reveal a serum bilirubin level of 2.9 mg/dL (49.6 µmol/L), a serum creatinine level of 1.3 mg/dL 91

Self-Assessment Test

Directions


Medical Knowledge Self-Assessment ProgramÂŽ

Hematology and Oncology

22 AMA PRA Category 1 Credits™ available until July 31, 2018.


Each of the numbered items is followed by lettered answers. Select the ONE lettered answer that is BEST in each case.

Hematology Questions Item 1 A 45-year-old woman is evaluated in the emergency department for a 1-day history of abdominal pain and fever. She also reports unexpected, heavy menstrual bleeding of 1 day’s duration and easy bruising of 2 days’ duration. Medical and family histories are unremarkable, and she takes no medications. On physical examination, the patient is oriented to person and place, but not time. Temperature is 38.1 °C (100.6 °F), blood pressure is 170/98 mm Hg, pulse rate is 110/min, and respiration rate is 20/min. Other than confusion, neurologic examination is normal. Subconjunctival hemorrhages are present. Cardiopulmonary examination is normal. Abdominal examination reveals tenderness to palpation without guarding or rebound. Pelvic examination shows blood in the vaginal vault with no cervical motion tenderness or adnexal masses. Laboratory studies: Hematocrit Leukocyte count Platelet count Reticulocyte count Bilirubin, total Creatinine Lactate dehydrogenase

26% 10,300/µL (10.3 × 109/L) 24,000/µL (24 × 109/L) 8.3% of erythrocytes 2.3 mg/dL (39.3 µmol/L) 3.2 mg/dL (283 µmol/L) 1500 U/L

Which of the following is the most appropriate diagnostic test to perform next? (A) (B) (C) (D)

ADAMTS-13 activity level Osmotic fragility test Peripheral blood smear Stool Shiga toxin assay

Item 2 A 75-year-old man arrives at the emergency department after passing three large-volume, melenic stools over a 2-hour period. Medical history is significant for atrial fibrillation and hypertension. Medications are warfarin, metoprolol, and lisinopril. On physical examination, he is diaphoretic and the skin is cool to the touch. Temperature is 36.8 °C (98.2 °F), blood pressure is 82/64 mm Hg, pulse rate is 142/min and irregular, and respiration rate is 20/min. Oxygen saturation is 95% breathing ambient air. Cardiac examination reveals tachycardia. Pulmonary examination is normal. Peripheral pulses are thready. Rectal examination reveals melenic stool that is guaiac positive. Laboratory studies: Hemoglobin Leukocyte count Platelet count INR

8.2 g/dL (82 g/L); 12.8 g/dL (128 g/L) 3 months ago 8600/µL (8.6 × 109/L) 183,000/µL (183 × 109/L) 7.4

In addition to intravenous vitamin K and fluid resuscitation, which of the following is the most appropriate treatment? (A) 4-Factor prothrombin complex concentrate (B) Activated factor VII

(C) Cryoprecipitate (D) Fresh frozen plasma

Item 3 A 37-year-old woman is evaluated for a 6-month history of progressive shortness of breath. Although she remains physically active, she becomes dyspneic when walking up multiple flights of stairs or running to catch a bus. Medical history is significant for a diagnosis of a pulmonary embolism 2 years ago, which was associated with oral contraceptive use. She was initially treated with low-molecular-weight heparin followed by therapeutic warfarin for 3 months. She is a nonsmoker. Medical history is otherwise unremarkable, and she takes no medications. On physical examination, she is afebrile, blood pressure is 128/76 mm Hg at rest, pulse rate is 72/min, and respiration rate is 15/min. Oxygen saturation is 98% breathing ambient air. Pulmonary examination reveals clear lungs. Cardiac examination is significant for a fixed, split S2, a holosystolic murmur at the left sternal border that increases on inspiration, and a heave. Trace lower extremity bilateral edema is present. The remainder of the examination is noncontributory. Walking up stairs at the office at a moderate pace, she becomes short of breath after two flights of stairs, oxygen saturation decreases to 92%, and pulse rate increases to 145/min. A chest radiograph is normal, showing no parenchymal abnormalities. Transthoracic echocardiography shows right atrial and ventricular dilation and moderate tricuspid regurgitation but no other valvular abnormalities. Which of the following is the most appropriate diagnostic test to perform next? (A) (B) (C) (D)

Pulmonary CT angiography Serum D-dimer test Venous Doppler ultrasonography of the legs Ventilation-perfusion (V/Q) lung scan

Item 4 A 48-year-old woman is evaluated for fatigue and intermittent abdominal discomfort of 2 months’ duration and occasional dark urine. Medical and family histories are unremarkable. Her only medication is an oral contraceptive pill. On physical examination, temperature is 37.2 °C (99.0 °F), blood pressure is 125/74 mm Hg, pulse rate is 68/min, and respiration rate is 13/min. Pallor is observed, and abdominal tenderness is present on palpation. No icterus, bruising, or splenomegaly is noted. Laboratory studies: Hemoglobin Leukocyte count Platelet count Reticulocyte count Bilirubin, total Direct antiglobulin (Coombs) test

7.2 g/dL (72 g/L) 3000/µL (3 × 109/L) with a normal differential 125,000/µL (125 × 109/L) 8% of erythrocytes Normal Negative 129

Self-Assessment Test

Directions


Answers and Critiques

Item 1

Answer:

C

Educational Objective: Diagnose thrombotic thrombocytopenic purpura. The most appropriate diagnostic test to perform at this time is a peripheral blood smear. This patient likely has thrombotic thrombocytopenic purpura (TTP). TTP should be suspected in patients who have microangiopathic hemolytic anemia, characterized by schistocytes on the peripheral blood smear and increased serum lactate dehydrogenase levels, and thrombocytopenia. A peripheral blood smear is essential to determine whether the anemia is caused by a microangiopathic hemolytic process as indicated by the presence of schistocytes. Patients may also have fever; kidney manifestations such as hematuria, elevated creatinine level, and proteinuria; and fluctuating neurologic manifestations, but the absence of these symptoms does not exclude the diagnosis. Assays for ADAMTS-13 activity and inhibitor titer are available but are best used for prognosis rather than to guide therapy, because TTP requires immediate treatment that cannot be delayed until laboratory test results are available. Low activity levels and a positive inhibitor titer confer a higher risk for relapse. An osmotic fragility test is used to evaluate for hereditary spherocytosis, which can produce hemolysis in the setting of an acute infection. However, hereditary spherocytosis does not cause thrombocytopenia, kidney injury, or mental status changes. TTP can overlap with hemolytic uremic syndrome (HUS), which usually occurs in children. HUS may be precipitated by an infectious diarrheal illness, especially Escherichia coli O157:H7 or Shigella species. These bacteria elaborate a toxin that resembles antigens on renal endothelial cells and bind and cause renal cell death. It is not clinically helpful to attempt to distinguish between TTP and HUS, because many patients with HUS respond to plasma exchange, the treatment for TTP. Key PoinT

• Thrombotic thrombocytopenic purpura is a clinical diagnosis that requires the presence of thrombocytopenia and microangiopathic hemolytic anemia, which is confirmed by schistocytes on the peripheral blood smear.

Bibliography Crawley JT, Scully MA. Thrombotic thrombocytopenic purpura: basic pathophysiology and therapeutic strategies. Hematology Am Soc Hematol Educ Program. 2013;2013:292-9. [PMID: 24319194]

Item 2

Answer:

A

Educational Objective: Treat a patient for major bleeding who is taking warfarin. The patient should be given 4-factor prothrombin complex concentrate (4f-PCC) in addition to intravenous vitamin K and fluids. He is experiencing major bleeding complicated by warfarin therapy and requires immediate anticoagulation reversal. 4f-PCC is a plasma-derived product that contains all four vitamin K–dependent coagulation factors (factors II, VII, IX, and X). Unlike fresh frozen plasma (FFP), 4f-PCC is stored at room temperature, does not require ABO typing, and can be infused quickly because of its small volume, thus reducing the time to delivery of therapy. Compared with FFP, 4f-PCC has been shown to more rapidly achieve hemostasis in patients with visible or musculoskeletal bleeding with less risk of fluid overload and no difference in thromboembolic events. This agent has therefore been approved by the FDA for urgent reversal of coagulation factor deficiencies related to vitamin K antagonist therapy for adult patients with acute major bleeding, as well as for adult patients in need of urgent surgery or an invasive procedure. Activated factor VII (factor VIIa) has been evaluated in case series for the treatment of vitamin K antagonist– related bleeding. Although factor VIIa can correct the INR quickly in most instances, it is unclear if this is associated with achievement of optimal hemostasis considering factors II, IX, and X are not replaced with this agent. A low dose of factor VIIa may be used in conjunction with 3-factor PCCs (which contain very little factor VII) for treatment of major vitamin K antagonist–associated bleeding in situations when 4f-PCCs are not available and the patient has a contraindication to the use of FFP (for example, uncompensated heart failure). Cryoprecipitate is rich in fibrinogen and is used to treat inherited or acquired fibrinogen deficiency or dysfibrinogenemia. It has no role in the management of vitamin K antagonist–related bleeding. FFP can be used when 4f-PCC is not readily available. However, 4f-PCC is also less likely than FFP to induce transfusion-associated circulatory overload, an important consideration in patients with heart failure or transfusion-related acute lung injury. Furthermore, 4f-PCC goes through viral inactivation, which reduces the incidence of transfusion-transmitted infectious diseases. Key PoinT

• Major bleeding associated with vitamin K antagonists should be treated by reversing anticoagulation with 4-factor prothrombin complex concentrate in addition to intravenous vitamin K.

167

Answers and Critiques

Hematology Answers


Medical Knowledge Self-Assessment ProgramÂŽ

Neurology

16 AMA PRA Category 1 Credits™ available until July 31, 2018.


Headache and Facial Pain

in headache control. Although RCVS is sometimes complicated by stroke, seizures, or (rarely) death, most outcomes are benign. Posterior reversible encephalopathy syndrome (PRES) is a disorder of cerebrovascular regulation frequently showing clinical and neuroimaging overlap with RCVS. Headaches are common but not necessarily thunderclap in type. Nausea, vomiting alterations in mental status, seizures, and visual compromise are all frequently noted. Brain MRI reveals areas of white matter edema in posterior brain regions (occipital and parietal cortex), and vasoconstriction may not be present. Causes include malignant hypertension, eclampsia, and various drugs, such as sympathomimetic agents and immunosuppressive drugs. Key Points

• Thunderclap headache requires emergent evaluation with head CT; if CT scans are normal, lumbar puncture is mandatory. • Most (85%) thunderclap headaches resulting from subarachnoid hemorrhage involve rupture of previously unidentified saccular aneurysms.

HVC

• No evidence supports the superiority of anticoagulation over antiplatelet therapy in prevention of stroke after carotid and vertebral artery dissections, and most patients are initially managed with aspirin.

Idiopathic Intracranial Hypertension (Pseudotumor Cerebri) Idiopathic intracranial hypertension (IIH), also called pseudotumor cerebri, is characterized by increased intracranial pressure without identifiable structural pathology. This disorder may arise from elevated venous pressure and secondarily increased resistance to CSF absorption. More than 90% of affected patients are female, obese, and of child-bearing age. Other risk factors include hypervitaminosis A, tetracycline antibiotics, isotretinoin, pregnancy, and glucocorticoid use or withdrawal. Headaches are typically diffuse, steady or throbbing, and worse in the mornings or with the Valsalva maneuvers. Brief episodes of bilateral dimming of vision lasting seconds (“visual obscurations”) are classically reported, but blurring, scotomas, and diplopia also are noted. Pulsatile tinnitus, dizziness, and neck pain are common. Papilledema is almost always present on examination. Diagnosis is confirmed by a CSF opening pressure greater than 250 mm H2O with normal fluid composition. Brain MRI may either be normal or show small ventricles, a partially empty sella turcica, widening of the optic nerve sleeves, or flattening of the optic globes. Visual field testing is crucial in the initial and follow-up evaluations of IIH. Enlargement of the blind spot and reduction in peripheral fields are commonly seen, and patients who do not improve with standard treatment may require surgical methods that reduce CSF pressure along the optic nerve to preserve vision. 4

The carbonic anhydrase inhibitor acetazolamide is considered the drug of choice to treat this disorder. Topiramate possesses a weak carbonic anhydrase effect but may provide the added benefit of weight loss. In patients with an elevated BMI, a weight-reduction program is necessary, with some requiring bariatric procedures. Repeated lumbar punctures may be helpful in the early stages of the disorder as medication takes effect and in the settings of pregnancy or poor surgical risk. CSF decompressive procedures (such as optic nerve fenestration) or lumboperitoneal shunting should be considered when medical options are ineffective. Key Point

• Carbonic anhydrase inhibitors, such as acetazolamide or topiramate, are generally used to treat idiopathic intracranial hypertension.

Headaches from Intracranial Hypotension Headaches from intracranial hypotension are characteristically postural, worsening in the upright and improving in the supine position. Some affected patients may report dull interval head pain, neck pain or stiffness, diplopia, nausea, vertigo, tinnitus, or hyperacusis. Physical examination findings are usually normal, although sixth nerve palsy or other abnormalities are possible. CSF hypotension may arise as a result of lumbar puncture, surgery, or trauma or may occur spontaneously. Risk factors for postdural puncture headaches include young age, female sex, low BMI, and the use of larger gauge or conventional cutting (traumatic) needles. Contrary to previous reports, a migraine history does not appear to increase the risk. Evaluation of patients with spontaneous intracranial hypotension is initiated with contrast-enhanced MRI scans of the brain and spinal cord. Brain MRI can be normal or reveal diffuse pachymeningeal thickening and enhancement, subdural fluid collections, or caudal cerebellar tonsillar descent suggestive of a Chiari malformation. If the spinal MRI does not reveal the site of leak, CT myelography should then be performed. Conservative treatment measures for all forms of intracranial hypotension include bed rest, analgesia, and fluid resuscitation. In patients who do not respond to conservative measures, 10 to 15 mL of homologous blood can be injected into the epidural space. The resultant blood patch is associated with resolution of symptoms in 80% to 90% of patients. Alternative treatments include intravenous caffeine and epidural saline infusions, local injections of fibrin glue, and surgical correction of the dural tear or diverticula. Key Point

• Epidural blood patching is highly effective in resolving headaches from cerebrospinal fluid leaks that failed to respond to conservative management.


Each of the numbered items is followed by lettered answers. Select the ONE lettered answer that is BEST in each case.

Item 1

Item 3

A 42-year-old man is evaluated in the emergency department for a 1-week history of bilateral leg weakness and numbness. He has an 8-year history of multiple sclerosis (MS) that is currently well controlled with natalizumab; he has had no MS exacerbations since beginning treatment 2 years ago after unsuccessful trials of interferon beta and glatiramer acetate. The patient also has chronic fatigue and depression. Medications are monthly natalizumab, twice daily amantadine and extended-release bupropion, a daily multivitamin, and a calcium–vitamin D supplement that he rarely takes. On physical examination, temperature is 36.7 °C (98.1 °F), blood pressure is 124/58 mm Hg, pulse rate is 74/min, and respiration rate is 14/min. Muscle strength is 4/5 in the bilateral hip flexors, knee flexors, and foot dorsiflexors. Decreased pinprick sensation is noted just below the umbilicus. Laboratory studies performed 3 weeks ago showed no evidence of elevated serum antibody titers against the JC virus. Results of current complete blood count, liver chemistry studies, and a urinalysis show no abnormalities. An MRI of the brain shows white matter hyperintensities consistent with MS and is unchanged from an MRI obtained 1 year ago.

A 35-year-old man is evaluated in the emergency department for a 7-hour history of midback pain and bilateral leg numbness. He was in a bar fight immediately before symptom onset and sustained forceful kick injuries to the back, head, and limbs; he did not lose consciousness. On physical examination, temperature is 36.6 °C (97.8 °F), blood pressure is 110/70 mm Hg, pulse rate is 108/ min, and respiration rate is 18/min; BMI is 32. The patient is alert without any apparent cognitive deficits. Lacerations on the face, scalp, and extremities are noted, as are hematomas on the midback and chest. No tremors or significant swelling or hematomas on the scalp are detected. Muscle strength is 3/5 in the lower extremities, and muscle tone in the legs is reduced. Muscle tone in the arms is normal, and anal sphincter tone is reduced. Pinprick testing shows a sensory level below T8.

In addition to a 5-day infusion of intravenous methylprednisolone, which of the following is the most appropriate next step in management? (A) (B) (C) (D)

Discontinuation of natalizumab Measurement of serum 25-hydroxyvitamin D level MRI of the lumbar spine Oral trimethoprim-sulfamethoxazole for 5 days

Item 2 A 67-year-old woman is evaluated for a 1-year history of increasing forgetfulness. She reports greater difficulty keeping track of upcoming appointments, recalling details of recent telephone conversations, and remembering names of new acquaintances. She has completed 16 years of formal education, currently works as a teacher’s assistant, and has noticed no change in her ability to perform classroom duties, including carrying out the instructions of the teachers with whom she works. The patient lives alone and is able to care for herself, drive, and manage her finances. She describes her mood as “upbeat,” continues to enjoy her life, and has had no other symptoms. She does not take any medication. On physical examination, vital signs are normal. All other physical examination findings, including those from a neurologic examination, are normal. She scores 24/30 on the Montreal Cognitive Assessment, losing points in the orientation and delayed recall sections. Which of the following is the most likely diagnosis? (A) (B) (C) (D)

Dementia Depression Mild cognitive impairment Normal aging

Which of the following is the most appropriate next step in management? (A) (B) (C) (D)

CT of the head High-dose methylprednisolone MRI of the thoracic spine Phenytoin

Item 4 A 55-year-old man is evaluated in the emergency department for a 20-minute episode of left eye visual loss without pain followed by a 5-minute episode of slurred speech. He has no residual symptoms. The patient has hypertension treated with amlodipine. He takes no other medication. On physical examination, blood pressure is 178/92 mm Hg, pulse rate is 78/min and regular, and respiration rate is 12/min. Carotid upstrokes are normal without bruits. Heart rate is regular, and no murmurs are heard. Other physical examination findings, including those from a neurologic examination, are normal. Findings on an electrocardiogram and a noncontrast CT scan of the head are normal. Which of the following is the most appropriate next diagnostic test? (A) (B) (C) (D)

Carotid ultrasonography CT angiography of the neck MRI of the brain Transesophageal echocardiography

Item 5 A 42-year-old man comes to the office to discuss results of imaging studies, which were ordered because of a change in his pattern of chronic migraine. Headache episodes have now improved. On physical examination, temperature is normal, blood pressure is 110/80 mm Hg, pulse rate is 80/min, and respiration rate is 16/min. All other findings from the general physical and neurologic examinations are normal. 91

Self-Assessment Test

Directions


Answers and Critiques Answer:

B

Educational Objective: Diagnose vitamin D deficiency in a patient with multiple sclerosis. This patient’s serum 25-hydroxyvitamin D level should be measured. He most likely has experienced a breakthrough relapse of multiple sclerosis (MS), which should be treated with intravenous methylprednisolone. Given his 2 years without relapses, the MS is relatively well-controlled. However, his care is not fully optimized because he rarely takes the vitamin D supplement. Accumulating evidence suggests that disease activity in MS is highly linked with serum vitamin D levels, with less frequent relapses and fewer new MRI lesions in patients with higher levels. This patient’s serum 25-hydroxyvitamin D level should thus be measured to determine if he is vitamin D deficient. Vitamin D supplementation as an adjunctive treatment in MS has been shown to be superior to disease-modifying therapy alone and has become a standard of care for MS patients, especially those who are vitamin D deficient, although the ideal dosing regimen and serum 25-hydroxyvitamin D level are still unknown. Pharmacodynamic studies are under way that may help inform dosing regimens. Vitamin D supplementation also protects against osteoporosis, for which patients with MS are at higher risk. This patient has no clear indications supporting discontinuation of natalizumab at this time. Natalizumab is the most highly effective drug for MS currently available, which is borne out by his relapse-free status for 2 years and stable brain MRI results. Although this drug is sometimes associated with progressive multifocal leukoencephalopathy (PML), his recent negative result on JC virus antibody testing place him at a very low risk for development of PML, and the spinal cord localization of this current relapse also argues against PML, which does not affect the spinal cord. The bilateral leg weakness and sensory level around the umbilicus (T10) are consistent with localization to the thoracic spinal cord. Therefore, an MRI of the lumbar spine would not be appropriate. This patient has no indication for antibiotic therapy, which makes a 5-day course of trimethoprim-sulfamethoxazole inappropriate. Key PoinT

• Vitamin D supplementation as an adjunctive treatment in multiple sclerosis (MS) has been shown to be superior to disease-modifying therapy alone and has become a standard of care for patients with MS, especially those who are vitamin D deficient.

Bibliography Golan D, Halhal B, Glass-Marmor L, et al. Vitamin D supplementation for patients with multiple sclerosis treated with interferon-beta: a randomized

controlled trial assessing the effect on flu-like symptoms and immunomodulatory properties. BMC Neurol. 2013 Jun 14;13:60. [PMID: 23767916]

Item 2

Answer:

C

Educational Objective: Diagnose mild cognitive impairment. The most likely underlying cause of this patient’s symptoms is mild cognitive impairment (MCI). MCI is a cognitive state between normal aging and dementia characterized by a decline in cognitive functioning that is greater than what is expected with normal aging but has not resulted in significant functional disability. For most patients, the onset is insidious, and for some, the course may be progressive; 10% to 15% of patients with MCI transition to dementia per year, compared with 1% to 2% per year of the general population. The Montreal Cognitive Assessment is a screening tool that is more sensitive than the Mini–Mental State Examination in the detection of MCI because it has more cognitively challenging tests of memory/recall and executive function. A score lower than 26/30 generally suggests cognitive impairment, especially in patients with 16 years of formal education. In clinical practice, a careful history and results of a standard mental examination are often sufficient to make a diagnosis of MCI, and extensive cognitive testing is not routinely required. Occasionally, a formal battery of neuropsychological testing beyond the standard mental examination is needed to distinguish particularly mild cases of cognitive impairment from normal aging. In order to meet criteria for dementia, a patient’s cognitive deficits must interfere with daily functioning and result in some loss of independence. A detailed history of the patient’s abilities to perform activities of daily living, such as paying bills, managing financial records, assembling tax records, shopping alone, working on hobbies, taking medications, driving, and remembering recent holidays or family events, should be obtained to elicit any change in function. This patient does not meet the criteria for dementia. The diagnosis of clinical depression is based on patient history and exclusion of alternative diagnoses; no additional tests can confirm the diagnosis. The evaluation must establish whether the patient meets established criteria for major depression, dysthymia, or a different psychiatric condition and also assess for substance abuse. Depressed mood and anhedonia are cardinal symptoms, and the presence of either is highly sensitive but not specific for major depression. Using a two-item questionnaire that assesses for the presence of depressed mood or anhedonia is a quick way to screen for depression. If either depressed mood or anhedonia is present, further inquiry or employing a second tool to diagnose depression should be pursued. This patient, who describes her mood as upbeat and says she enjoys her 115

Answers and Critiques

Item 1


Answers and Critiques

life, has neither depressed mood nor anhedonia. Therefore, depression is unlikely to be the cause of her symptoms. Patients with memory problems due to normal aging have symptoms, most notably memory loss, that are commonly associated with cognitive impairment, but cognitive testing shows functioning within the normal range. This patient’s memory difficulties are greater than what is expected with normal aging, and her score on the Montreal Cognitive Assessment is not in the normal range. Key PoinT

Answers and Critiques

• A score lower than 26/30 on the Montreal Cognitive Assessment generally suggests cognitive impairment, especially in patients with many years of formal education.

Bibliography Petersen RC. Clinical practice. Mild cognitive impairment. N Engl J Med. 2011 Jun 9;364(23):2227-34. [PMID: 21651394]

Item 3

Answer:

B

Educational Objective: Treat acute spinal cord injury. This patient should receive high-dose methylprednisolone as the next step in management. He has an acute spinal cord injury most likely due to traumatic fracture of a thoracic vertebra and subsequent spinal cord compression. The localization of the injury at approximately the T8 level of the spinal cord is clear, given the bilateral leg weakness and reduced tone, reduced anal sphincter tone, and the sensory level on pinprick testing. Large clinical trials have shown improved motor function recovery up to 1 year after administration of an intravenous bolus of methylprednisolone, 30 mg/kg, within the first 8 hours of traumatic spinal cord injury followed by a 5.4-mg/kg infusion over the next 23 hours. A recent trial has shown that extending this infusion for an additional 24 hours further increases recovery. Because of these studies, immediate administration of high-dose methylprednisolone for suspected traumatic spinal cord injury has become standard of care. Diagnostic studies are not appropriate because of the acuity of the situation. Given that the patient experienced the trauma 7 hours before he was seen in the emergency department, ordering CT or MRI would delay initiation of treatment, which is necessary within the first 8 hours of the traumatic event. Obtaining a confirmatory MRI of the thoracic spine at a later time is appropriate. However, CT of the head is not needed because the injury localizes to the thoracic spine on examination. Although the patient received some trauma to the head, CT is likely unnecessary, given that he had no loss of (or impaired) consciousness, no evidence of significant external head trauma, and no other signs or symptoms of traumatic brain injury. Phenytoin would not treat this patient’s spinal cord injury. Administration of phenytoin after significant head trauma may be indicated to prevent seizures, but this patient does not appear to have significant head trauma, nor is this the most acute issue at this time. 116

Key PoinT

• High-dose methylprednisolone administered within 8 hours of a traumatic spinal cord injury has been shown to improve motor function recovery.

Bibliography Bracken MB. Steroids for acute spinal cord injury. Cochrane Database Syst Rev. 2012 Jan 18;1:CD001046. [PMID: 22258943]

Item 4

Answer:

A

Educational Objective: Evaluate transient ischemic attack. The patient should undergo carotid ultrasonography. He most likely has experienced a transient ischemic attack (TIA), which implies the absence of retinal or cerebral infarction. His ABCD2 score, which is based on a patient’s Age, Blood pressure, Clinical presentation, Duration of symptoms, and the presence of Diabetes mellitus, is 2 (one point for elevated blood pressure and one point for the symptom of slurred speech), which indicates a 2-day stroke risk of 1.3%. The antecedent transient monocular blindness in the left eye is concerning for extracranial atherosclerosis of the internal carotid artery. Hospital admission is recommended for all patients with TIAs who have an ABCD2 score of 3 or greater to expedite diagnostic testing and stroke subtyping; admission is also recommended for patients with a score of 0 to 2 if rapid outpatient evaluation cannot be performed. Carotid ultrasonography to evaluate for symptomatic extracranial internal carotid artery stenosis is the most appropriate next diagnostic test in this patient with a TIA, given the high risk of early recurrence. Patients with greater than 70% extracranial internal carotid artery atherosclerotic stenosis have the highest risk of stroke in the 2 weeks after a TIA. Carotid Duplex ultrasonography is noninvasive and can effectively rule out significant atherosclerotic disease. If the ultrasound suggests greater than 50% stenosis, hospital admission and a confirmatory test with magnetic resonance or CT angiography is appropriate, with plans for early revascularization. Rapid cardiac testing with transthoracic echocardiography and cardiac rhythm evaluation also is advised within 24 hours for all patients with suspected TIA, as is vascular imaging of the extracranial carotid arteries. CT angiography of the neck is inappropriate at this point because extracranial internal carotid artery stenosis can be excluded without exposing the patient to a highly invasive procedure with contrast and radiation. Although an MRI of the brain can distinguish a TIA from an ischemic stroke and reveal infarcts in other arterial territories, it is inappropriate as the next diagnostic test in this patient because results are unlikely to affect immediate management. In addition, MRI may not be readily available and may be contraindicated in some patients. Transesophageal echocardiography may be indicated to identify embolic sources of a TIA or stroke in patients in whom noninvasive diagnostic testing has been unrevealing. However, the yield of transesophageal echocardiography is


Medical Knowledge Self-Assessment ProgramÂŽ

Rheumatology

16 AMA PRA Category 1 Credits™ available until July 31, 2018.


approach to the Patient with Rheumatologic Disease

pain with both passive and active range of motion implies an intrinsic joint condition, whereas pain with only active range of motion may be due to a periarticular condition. See Musculoskeletal Pain in MKSAP 17 General Internal Medicine for more information. Key Points

HVC

• A physical examination is essential when diagnosing musculoskeletal pain and can help to avoid unnecessary laboratory and radiographic testing. • Pain with both passive and active range of motion implies an intrinsic joint condition, whereas pain with only active range of motion may be due to a periarticular condition.

noninflammatory oligoarthritis, particularly if the involved joints share a history of trauma or overuse.

Polyarthritis Polyarthritis involves five or more joints. The most common causes of chronic inflammatory polyarthritis include rheumatoid arthritis, systemic lupus erythematosus (SLE), and psoriatic arthritis (see Table 2). In contrast, acute onset of inflammatory polyarthritis is more commonly caused by viral infections such as hepatitis, parvovirus, rubella, herpes, HIV, adenovirus, mumps, or enterovirus. Polyarthritis may also be caused by a drug-induced serum sickness reaction, an immune complex reaction to bacterial infections such as endocarditis, or other forms of crystal or autoimmune diseases. Key Point

arthritis In addition to the presence or absence of inflammation, other features help to refine the differential diagnosis of musculoskeletal pain: pattern of joint involvement, number of joints involved, duration of symptoms, and presence or absence of symmetry.

Monoarticular arthritis Monoarticular arthritis (monoarthritis) involves a single joint. The differential diagnosis depends upon whether there is inflammation, the acuity of symptoms, and the patient’s demographic and epidemiologic risk factors. Inflammatory monoarthritis may be due to infection: bacterial in acute cases, and atypical organisms such as fungi, mycobacteria, or spirochetes (Lyme) in chronic cases. Noninfectious causes include crystal-related and autoimmune diseases. Crystal-related diseases typically present acutely but occasionally can be chronic, especially those caused by calcium pyrophosphate deposition. Noninflammatory monoarthritis is usually due to osteoarthritis or mechanical derangement (such as a torn meniscus or ligament). A history of trauma prior to onset of symptoms points toward noninflammatory arthritis. The most appropriate and cost-effective means of assessing the cause of acute monoarthritis is by aspiration and analysis of the synovial fluid for leukocytes, Gram stain with culture, and crystals. If there is high suspicion for infection (especially atypical organisms) and synovial fluid cultures are unrevealing, synovial biopsy or other special tests may be needed. Blood in the synovial fluid (hemarthrosis) is typically associated with trauma except in patients with bleeding diatheses such as hemophilia.

• The most appropriate and cost-effective means of assessing the cause of acute monoarthritis is by aspiration and analysis of the synovial fluid for leukocytes, Gram stain with culture, and crystals.

extra-articular Manifestations of Rheumatologic Disease Many rheumatologic diseases are systemic and therefore cause extra-articular manifestations in various sites and organs.

Constitutional Symptoms Constitutional symptoms are common in systemic inflammatory diseases. Fever is especially typical of adult-onset Still disease and the autoinflammatory diseases (also known as the periodic fever syndromes) but may also occur in SLE, rheumatoid arthritis, and vasculitis, in which it is commonly low grade. Malaise and unintended weight loss also occur. In the absence of a clear cause, such symptoms may also suggest occult malignancy and/or infection.

Dermatologic Manifestations Skin involvement is common in many rheumatologic diseases (Table 3, on page 4). Because of easy access by physical examination and/or biopsy, skin manifestations can be helpful in establishing a diagnosis, especially SLE, systemic sclerosis, and vasculitis. Some cutaneous findings may be subtle or unnoticed by the patient (for example, nail pitting); therefore, a high index of suspicion and a thorough physical examination may be required.

Oligoarthritis

Inflammatory eye Disease

Oligoarthritis involves two to four joints, commonly in the lower extremities, and is often asymmetric. Although many diseases can manifest with oligoarthritis, the most common autoimmune inflammatory forms of oligoarthritis are the spondyloarthritis diseases (Table 2). Disseminated gonococcal infection, rheumatic fever, and Lyme disease may also present as inflammatory oligoarthritis. Osteoarthritis may cause a

Eye involvement is common in systemic rheumatologic diseases and provides direct visual access to both mucosal and central nervous system tissues. The location and type of ocular involvement may help narrow the differential diagnosis (Table 4, on page 4). Some diseases are particularly notable for ocular involvement, including rheumatoid arthritis (episcleritis and scleritis); spondyloarthritis and sarcoidosis (uveitis);

2

HVC


Self-Assessment Test

Self-Assessment Test

Which of the following is the most likely diagnosis? (A) (B) (C) (D)

Amyopathic dermatomyositis Polymorphous light eruption Rosacea Systemic lupus erythematosus

Item 5 A 62-year-old woman is evaluated for a 2-year history of progressively frequent and severe pain in the right knee. She has osteoarthritis with good control of her other joint symptoms with her current therapy that includes medication and a daily exercise regimen. She notes about 20 minutes of morning stiffness in the right knee with significant pain with use after rest; her activities are increasingly limited due to these symptoms. History is otherwise unremarkable. Medications are acetaminophen and celecoxib. On physical examination, blood pressure is 135/82 mm Hg. BMI is 32. There are Heberden nodes of the second and fifth distal interphalangeal joints bilaterally and Bouchard nodes of the second and third proximal interphalangeal joints bilaterally. Bony hypertrophy of the knees is present. There is a positive bulge sign for effusion of the right knee with slight warmth but no erythema. Standing radiographs of the knees show right (greater than left) medial joint-space narrowing, bilateral osteophytes, and bilateral peaking of the tibial spines. Aspiration of the right knee is performed; synovial fluid analysis shows a leukocyte count of 250/µL (0.25 × 109/L) and no evidence of crystals. Which of the following is the most appropriate next step in management? (A) (B) (C) (D)

Administer intra-articular glucocorticoids Administer intra-articular hyaluronic acid Refer for arthroscopic lavage Substitute indomethacin for celecoxib

Item 6 A 72-year-old man is evaluated in the emergency department for acute onset of pain and swelling of the left knee. He was diagnosed with community-acquired pneumonia 4 days ago, and a 7-day course of clarithromycin was started at that time. He reports marked improvement of his respiratory symptoms. History is also significant for gout, with attacks occurring approximately once a year; hypertension; diet-controlled diabetes mellitus; and chronic kidney disease. Other medications are nifedipine and hydrochlorothiazide. On physical examination, temperature is 37.1 °C (98.8 °F), blood pressure is 117/86 mm Hg, pulse rate is 76/min, and respiration rate is 14/min. BMI is 32. Mildly decreased breath sounds in the right lung midfield are noted. The left knee is swollen, red, warm, tender, and fluctuant with limited range of motion. Laboratory studies are significant for a leukocyte count of 7200/µL (7.2 × 109/L) and a serum creatinine level of 1.7 mg/dL (150.3 µmol/L). 94

A radiograph of the left knee is normal. Aspiration of the left knee is performed; synovial fluid analysis reveals a leukocyte count of 20,000/µL (20 × 109/L), extracellular and intracellular urate crystals, and a negative Gram stain. Which of the following is the most appropriate treatment? (A) (B) (C) (D)

Acetaminophen Colchicine Indomethacin Intra-articular glucocorticoids

Item 7 A 65-year-old man is evaluated for severe abdominal pain, joint pain, and a rash. He states that he had an upper respiratory infection about 10 days ago. Three days ago he noted a rash on his lower extremities. One day later, he experienced pain in his knees and ankles, along with abdominal pain that worsened over the past two days. He reports no visual symptoms, numbness, weakness, or other symptoms. On physical examination, the patient appears uncomfortable. The chest and cardiac examinations are unremarkable. Decreased bowel sounds and diffuse abdominal tenderness without rebound are noted. The knees and ankles are tender and mildly swollen. Palpable purpuric lesions are present on the lower extremities, including the soles of the feet. The remainder of the physical examination reveals no abnormalities. Laboratory studies show a normal complete blood count, an erythrocyte sedimentation rate of 88 mm/h, a serum creatinine level of 1.7 mg/dL (150.3 µmol/L), and a urinalysis showing 3+ protein, 20-30 erythrocytes/hpf, 20-30 leukocytes/hpf, and mixed granular and cellular casts. A stool test is positive for occult blood. An abdominal ultrasound reveals thickening and edema of the ileum. A biopsy of an affected skin lesion demonstrates the presence of small-vessel, leukocytoclastic vasculitis accompanied by deposition of IgA. Which of the following is the most appropriate therapy at this time? (A) (B) (C) (D)

Cyclophosphamide Dapsone Ibuprofen Prednisone

Item 8 A 28-year-old woman is evaluated for a 6-month history of joint pain and swelling. She was diagnosed with rheumatoid arthritis 5 years ago; current medications are etanercept, sulfasalazine, and etodolac. She was initially treated with methotrexate, which was stopped due to gastrointestinal intolerance, and she refuses to retry it. On physical examination, temperature is 36.7 °C (98.0 °F), blood pressure is 126/74 mm Hg, pulse rate is 68/min, and respiration rate is 14/min. BMI is 24. Two proximal interphalangeal (PIP) joints of the left


Medical Knowledge Self-Assessment ProgramÂŽ

PART B Endocrinology and Metabolism General Internal Medicine Infectious Disease Nephrology Pulmonary and Critical Care Medicine

Part B AMA PRA Category 1 Credits™ available until December 31, 2018.


Medical Knowledge Self-Assessment ProgramÂŽ

Endocrinology and Metabolism

14 AMA PRA Category 1 Credits™ available until December 31, 2018.


Disorders of Glucose Metabolism

CONT.

meals composed of protein, fat, and carbohydrate to avoid the sensation of hypoglycemia.

acute Complications of Diabetes Mellitus Diabetic Ketoacidosis and Hyperglycemic Hyperosmolar Syndrome Diabetic ketoacidosis (DKA) and hyperglycemic hyperosmolar syndrome (HHS) are acute complications of uncontrolled hyperglycemia with life-threatening consequences if not recognized and treated early. DKA typically occurs in the setting of hyperglycemia with relative or absolute insulin deficiency and an increase in counterregulatory hormones. Sufficient amounts of insulin are not present to suppress lipolysis and oxidation of free fatty acids, which results in ketone body production and subsequent metabolic acidosis. DKA occurs more frequently with type 1 diabetes, although 10% to 30% of cases occur in patients with type 2 diabetes. HHS occurs in the setting of partial insulin deficiency that is more typical of type 2 diabetes. There is sufficient insulin in patients with HHS to suppress lipolysis and production of ketone bodies, but inadequate amounts to prevent the hyperglycemia, dehydration, and hyperosmolality characteristic of HHS. Several risk factors can precipitate the development of extreme hyperglycemia: infection, intentional or inadvertent insulin therapy nonadherence, myocardial infarction, stress, trauma, and confounding medications, such as glucocorticoids or atypical antipsychotic agents. In addition, DKA may be the initial clinical presentation in some patients with previously undiagnosed type 1 or type 2 diabetes. An illness or event that leads to dehydration will often precipitate the hyperglycemia associated with HHS. Symptoms of extreme hyperglycemia in DKA and HHS include polyuria, polydipsia, unintentional weight loss, vomiting, weakness, and mentation changes. Dehydration and metabolic abnormalities worsen as hyperglycemia progresses, which can lead to respiratory failure, lethargy, obtundation, coma, and death. DKA can occur within several hours of the inciting event. The development of HHS is less acute than DKA and may take days to weeks to develop. HHS typically presents with more extreme hyperglycemia and mental status changes compared with DKA. The initial evaluation of severe hyperglycemia includes serologic studies (plasma glucose, serum ketones, blood urea nitrogen, creatinine, electrolytes, calculated anion gap, arterial blood gases, osmolality, complete blood count with differential, blood cultures), urine studies (ketones, urinalysis, urine culture), chest radiograph, and an electrocardiogram. Urine and serum ketones are elevated in DKA; however, a negative measurement initially does not exclude DKA. β-Hydroxybutyrate is the major ketone body in DKA, but 14

ketone laboratory measurements often use the nitroprusside reaction, which only estimates acetoacetate and acetone levels that may not be elevated initially. Although hyperglycemia is the typical finding at presentation with DKA, patients can present with a range of plasma glucose values, including those in the normal range (Figure 1). The anion gap is elevated. Stress-related mild leukocytosis is often present. Higher levels of leukocytosis may indicate an infectious process as the etiology of the hyperglycemia. Serum sodium levels can be low due to osmotic shifts of water from the intracellular to extracellular spaces. Normal or elevated serum sodium levels are indicative of severe volume depletion. Serum potassium levels may be elevated due to shifts from the intracellular to extracellular spaces due to ketoacidosis and the absence of sufficient insulin. Normal or low potassium levels on presentation indicate low potassium stores in the body with need for correction prior to initiation of insulin therapy to avoid cardiac arrhythmias. Serum amylase and lipase levels also can be elevated in the absence of pancreatitis. HHS typically presents with normal or small amounts of urine or serum ketones. Plasma glucose values in HHS are typically greater than in DKA and can exceed 1200 mg/dL (66.6 mmol/L). The serum osmolality is elevated greater than 320 mOsm/kg H2O. The serum bicarbonate level is greater than 18 mEq/L (18 mmol/L), and the pH remains greater than 7.3. Treatment of DKA and HHS requires correction of hyperglycemia with intravenous insulin infusions, frequent monitoring and replacement of electrolytes, correction of hypovolemia with intravenous fluids, and possible correction of acidosis (Table 11). The ICU is the best place for management of severe hyperglycemia because of the frequent monitoring required with intravenous insulin therapy, the need for monitoring for potential electrolyte-induced arrhythmias, and the potential for rapid decompensation. Plasma glucose levels should be monitored initially every hour while on insulin infusion therapy. Electrolytes should be monitored every 2 to 4 hours, depending on the initial electrolyte deficits and level of acidosis. Key Points

• The development of hyperglycemic hyperosmolar syndrome is less acute than that of diabetic ketoacidosis and may take days to weeks to develop; however, hyperglycemic hyperosmolar syndrome typically presents with more extreme hyperglycemia and mental status changes compared with diabetic ketoacidosis. • Treatment of diabetic ketoacidosis and hyperglycemic hyperosmolar syndrome requires correction of hyperglycemia with intravenous insulin infusions, frequent monitoring and replacement of electrolytes, correction of hypovolemia with intravenous fluids, and possible correction of acidosis.


Medical Knowledge Self-Assessment ProgramÂŽ

General Internal Medicine

28 AMA PRA Category 1 Credits™ available until December 31, 2018.


General Internal Medicine High Value Care in Internal Medicine An economically unsustainable amount of money is spent on health care in the United States—18% of the U.S. gross domestic product—leaving less money to pay for other essential services such as public health and safety, infrastructure, and education. Despite spending more on health care than any other country, the United States has health outcomes, including mortality, survival, and life expectancy, that rank at or near the bottom when compared with other high-income countries. Misuse and overuse of medical interventions contribute significantly to this untenable health care spending. Physicians are uniquely positioned to take the lead in addressing these problems by partnering with patients and other health care providers to reduce the use of medical interventions (tests and treatments) that offer minimal or no benefit and may lead to unintended harm. High value care is care that balances the clinical benefit of a given medical intervention with its harms and costs, with the goal of improving patient outcomes. High value care represents a paradigm shift away from the belief that more care is better care toward the idea that evidence-based, individualized care is better care. High value care places patients and their outcomes, values, and concerns at the center of every major clinical decision, using cost-effective and low-risk tools (for example, history and physical examination) and patientcentered communication to improve patient outcomes. This approach helps lay a foundation for dealing with the psychological burden that accompanies diagnostic uncertainty and allows both patients and physicians to be more comfortable with a conservative course of care. New drugs, devices, procedures, and tests are the primary drivers of increased health care spending; however, it is critical that physicians use testing and medical technology judiciously and selectively assess whether potential benefits justify the costs. The Alliance for Academic Internal Medicine and the American College of Physicians (ACP) have developed a simple stepwise framework to help providers incorporate high value care into daily practice (Table 1). The ACP has also developed clinical recommendations and physician resources to help physicians practice high value care (available at http://hvc. acponline.org/index.html). Additionally, over 72 medical specialty societies and Consumer Reports (an independent product testing organization) have participated in the American Board of Internal Medicine Foundation’s Choosing Wisely campaign, which promotes stewardship of medical resources

Table 1. High Value Care Framework: Steps Toward High Value Care Step 1: Understand the benefits, harms, and relative costs of the interventions that you are considering. Step 2: Decrease or eliminate the use of interventions that provide no benefits and/or may be harmful. Step 3: Choose interventions and care settings that maximize benefits, minimize harms, and reduce costs (using comparative effectiveness and cost-effectiveness data). Step 4: Customize a care plan with patients that incorporates their values and addresses their concerns. Step 5: Identify system-level opportunities to improve outcomes, minimize harms, and reduce health care waste. Reprinted with permission from Smith CD; Alliance for Academic Internal Medicine– American College of Physicians High Value, Cost-Conscious Care Curriculum Development Committee. Teaching high-value, cost-conscious care to residents: the Alliance for Academic Internal Medicine–American College of Physicians Curriculum. Ann Intern Med. 2012 Aug 21;157(4):284-6. [PMID: 22777503]

by asking societies to create evidence-based lists of tests and procedures whose necessity should be questioned. The Choosing Wisely lists and accompanying patient educational information are available at www.choosingwisely.org. Key Point

• High value care is care that balances the clinical benefit of a given medical intervention with its harms and costs, with the goal of improving patient outcomes.

Interpretation of the Medical literature Introduction Physicians must be familiar with an ever-expanding knowledge base that is founded on published research. Consequently, physicians must understand the basic principles of research to be able to independently interpret the literature, remain current in their medical knowledge, and apply the results of studies to provide high value care for their patients.

Study Designs Many research study designs exist, and it is important to recognize the strengths and weaknesses of each and the appropriate application of information derived from these studies to clinical situations. 1


Interpretation of the Medical literature

experimental Studies CONT.

In an experimental study, subjects and interventions are determined at the study outset, and investigators and subjects are often blinded to the intervention to minimize biased outcomes. In the most common type of experimental study, a randomized controlled trial (RCT), subjects are randomly assigned to either an intervention or control group to equally distribute predictive factors and minimize confounding among the groups. Although RCTs are considered the most rigorous study design and have the greatest ability to draw causal inferences, these studies usually involve detailed protocols and patients with a narrow disease spectrum—conditions that are challenging to replicate in ordinary practice settings. Consequently, RCTs often have limited generalizability. A variation of an RCT is a clusterrandomized trial, in which groups (or clusters) of study subjects are randomized to a treatment or control group; this design is helpful in evaluating interventions directed toward specific groups of people instead of individual patients. A less rigorous experimental design is the quasi-experimental study, in which investigators assign patients to intervention and control groups in a nonrandom manner. This type of design is typically used when randomization would be impractical or unethical. The different types of experimental studies and observational studies are compared in Table 2.

Observational Studies An observational study does not employ interventions or patient assignment to groups; alternatively, researchers compare two or more naturally existing groups. Observational studies are often less rigorous than experimental studies, thus reducing the ability to draw causal inferences. Advantages of observational studies include the capacity to utilize natural practice settings and to involve patients with wider ranges of illnesses and exposures. A disadvantage of observational studies is that they are more subject to confounding and bias than experimental studies. Observational designs include cohort studies, casecontrol studies, cross-sectional studies, and case series. A cohort study investigates the outcomes of groups (cohorts) with or without certain exposures or treatments. An example is a study that examines the rates of type 2 diabetes mellitus among patients with high socioeconomic status versus patients with low socioeconomic status. In prospective cohort studies, patients are observed for outcomes going forward in time, whereas retrospective cohort studies look at patients’ histories, often after an outcome has occurred. Prospective cohort studies are more rigorous than retrospective cohort studies because they reduce bias by selecting patients and statistical methods a priori. The standard outcome measure for a cohort study is relative risk. A case-control study, which is usually retrospective, compares the outcomes of patients with a disease (cases) to those without a disease (controls). For instance, patients with and without type 2 diabetes could be compared with respect to exposure to high-calorie, fast-food diets. Case-control studies 2

can be particularly valuable in the study of rare diseases. To reduce bias in case-control studies, investigators carefully match selected cases to controls in terms of demographic and prognostic factors. Additionally, investigators often increase statistical power by recruiting more controls than cases. For casecontrol studies, the standard estimate of risk is the odds ratio. Cross-sectional studies examine associations between diseases and exposures within a group of patients at one point in time. This study design is most commonly used to determine disease prevalence and infer causation. Survey studies are generally cross-sectional studies. Case series include only patients with the condition of interest. These patients are evaluated, either prospectively or retrospectively, to identify exposures or outcomes. Cross-sectional and case series designs are limited, based on the absence of control groups.

Systematic Reviews Systematic reviews provide a comprehensive summary, synthesis, and analysis of the literature that pertains to a focused research question. Systematic reviews involve exhaustive literature searches, systematic data abstraction, multiple reviewers, and a narrative summary regarding the strengths and limitations of the analysis. Systematic reviews minimize error by combining results from many studies. They may also include meta-analysis, which involves the statistical analysis of pooled data from the studies identified in a systematic review meeting certain predefined criteria for adequacy. The purpose of meta-analysis is to draw conclusions using a greater amount of data than is available in each of the individual studies. The limitations of systematic reviews and meta-analysis result from variability in the identification and selection of studies and from the inclusion of weak studies. However, a systematic review with meta-analysis that is based on the rigorous selection of numerous, high-quality RCTs could be considered the highest level of evidence. Key Points

• In a randomized controlled trial, subjects are randomly allocated to an intervention or control group, and investigators and subjects are often blinded to the intervention to minimize biased outcomes. • Observational designs include cohort studies, casecontrol studies, cross-sectional studies, and case series; patients are not randomly allocated to an intervention or control group, thus reducing the ability to draw causal inferences. • A systematic review provides a comprehensive summary, synthesis, and analysis of the literature that pertains to a focused research question and may or may not include meta-analysis; systematic reviews minimize errors by combining results from many studies but may be limited by variability in the identification and selection of studies and from the inclusion of weak studies.


Routine Care of the Healthy Patient

number needed to treat (NNT), and if detrimental, the number needed to harm (NNH). Numbers needed are useful indicators of the clinical impact of an intervention because they provide a sense of magnitude expected from the intervention. Numbers needed are calculated by taking the reciprocal of the change in absolute risk. For example, for the two interventions previously discussed, the NNT in the first case is 5 (1/0.2) and in the second case 50 (1/0.02) (see Table 4). Key Points

• Confidence intervals provide more information than P values because they reveal the plausible range of an event, allowing clinical significance to be estimated. • Statistical significance does not equal clinical importance, especially for large studies with uncommon outcomes.

HVC

• Relative risk comparisons tend to exaggerate outcomes relative to absolute risk measures; whenever possible, absolute risk should be used when explaining risk to patients.

HVC

• Numbers needed are estimates of the number of patients who must receive an intervention to cause one patient to experience the outcome being studied; numbers needed are useful indicators of the clinical impact of an intervention by providing a sense of magnitude of benefit/harm expected from the intervention.

HVC

levels of evidence and Recommendations F i g u r e 1 . Nomogram for interpreting diagnostic test results. In this nomogram, a straight line drawn from a patient’s pretest probability of disease (which is estimated from experience, local data, or published literature) through the likelihood ratio for the test result will point to the posttest probability of disease. Reprinted with permission from Fagan TJ. Letter: Nomogram for Bayes theorem. N Engl J Med. 1975 Jul 31;293(5):257. [PMID: 1143310] Copyright 1975, Massachusetts Medical Society.

CONT.

rates of events, such as death or complications, in two study groups using measures that include relative risk, odds ratios, and hazard ratios (Table 4). Absolute comparisons, on the other hand, represent absolute (that is, total) differences in outcomes between two groups. A disadvantage of relative comparisons is the potential for exaggerated outcomes, especially if the outcomes are uncommon. For instance, interventions that reduce the rate of a disease from 40% to 20% and 4% to 2% each have a relative risk reduction of 50%. However, the absolute risk reduction (ARR) for the first case is 20%, whereas the ARR for the second case is 2%.

Numbers Needed Numbers needed are estimates of the number of patients who must receive an intervention to cause one patient to experience the outcome being studied; if beneficial, it is termed the 6

Physicians make clinical decisions about patients by interpreting evidence from the published literature; however, not all evidence is developed with the same rigor. Therefore, the U.S. Preventive Services Task Force has identified levels of evidence that reflect the rigor of methods used in a study (Table 5). Additionally, grades of recommendations for providing a clinical service were created by balancing the level of evidence with the risk versus benefit of the service (Table 6).

Routine Care of the Healthy Patient History and Physical examination Periodic Health examination Although the periodic health examination has been associated with increased delivery of preventive services, multiple studies have failed to show a beneficial effect of the periodic health examination on morbidity or mortality; however, many of these trials were older, limited in scope, and assessed out-of-date interventions. Many physicians argue that the periodic health examination builds physician-patient relationships, which may promote improved adherence to physician recommendations.


Medical Knowledge Self-Assessment Program®

Infectious Disease

19 AMA PRA Category 1 Credits™ available until December 31, 2018.


Infectious Disease Central Nervous System Infections Meningitis Viral Meningitis Causes Viral meningitis is the most common cause of “aseptic” meningitis, in which cerebrospinal fluid (CSF) bacterial cultures are negative. Most patients have typical meningitis symptoms, such as fever, nuchal rigidity, headache, and photophobia. A substantial proportion of viral meningitis is caused by enteroviruses, which commonly occur in the summer and fall in temperate climates when these organisms circulate in the environment. Enteroviral meningitis can also occur in winter and spring months; seasonal enteroviral infections are typically caused by coxsackievirus, echovirus, or other nonpolio enteroviruses. Herpes simplex virus (HSV) can also cause meningitis. These meningitis syndromes can be related to primary infections, with central nervous system (CNS) involvement as a secondary consequence, or reactivation of latent infection presenting as aseptic meningitis. Most patients with HSV meningitis associated with primary infection will have genital lesions. HSV type 2 (HSV-2) is more commonly associated with meningitis, whereas HSV type 1 (HSV-1) is associated with encephalitis. An association exists between Mollaret meningitis, a benign recurring form of lymphocytic meningitis, and HSV-2; at least 84% of patients have evidence of HSV-2 in CSF. Primary HIV infection can present with an aseptic meningitis syndrome, which may occur in isolation from the other symptoms. Meningitis due to acute HIV infection is typically self-limited and may be clinically indistinguishable from other viral meningitis syndromes. Mumps virus can cause meningitis, with typical symptoms of fever, headache, and neck stiffness. Since the advent of universal childhood vaccination for measles, mumps, and rubella, the incidence of mumps-related meningitis has dramatically decreased. Meningitis from mumps virus can occur at any point during the course of clinical mumps infection. Parotitis or orchitis may be present. Infections with arboviruses, such as West Nile virus or St. Louis encephalitis virus, can cause meningitis, although these infections more typically produce encephalitis. Cytomegalovirus, Epstein-Barr virus, adenovirus, and varicella virus infections can also cause meningitis.

Diagnosis Symptoms of viral meningitis include fever, headache, neck stiffness, photophobia, and change in mental status and may be indistinguishable from symptoms of bacterial meningitis. Characteristics of the CSF profile are outlined in Table 1. Additional physical examination findings (rash and pharyngitis in acute HIV infection; parotitis or orchitis in mumps-related meningitis) can support the presumptive diagnosis. Polymerase chain reaction (PCR) and other molecular tests are available for diagnosis of numerous viral causes. CSF PCR studies may be used for diagnosing HSV and enterovirus meningitis; antibody detection in CSF is preferred for West Nile virus meningitis. Viral cultures of CSF are not clinically useful because of poor sensitivity and long turn-around times. Serologic testing may be an adjunctive diagnostic tool in meningitis caused by mumps.

Table 1. Typical CSF Findings in Patients with Viral and Bacterial Meningitis CSF Parameter

Viral Meningitisa

bacterial Meningitis

Opening pressure

≤250 mm H2O

200-500b mm H2O

Leukocyte count

50-1000/µL (50-1000 × 106/L)

1000-5000/µL (1000-5000 × 106/L)c

Leukocyte predominance

Lymphocytesd

Neutrophils

Glucose

>45 mg/dL (2.5 mmol/L)

<40 mg/dL (2.2 mmol/L)e

Protein

<200 mg/dL (2000 mg/L)

100-500 mg/dL (1000-5000 mg/L)

Gram stain

Negative

Positive in 60%-90%f,g

Culture

Negative

Positive in 70%-85%g

CSF = cerebrospinal fluid. aPrimarily

nonpolio enteroviruses (echoviruses and coxsackieviruses).

bValues

exceeding 600 mm H2O suggest the presence of cerebral edema, intracranial suppurative foci, or communicating hydrocephalus.

cRange

may be <100/µL (100 × 106/L) to >10,000/µL (10,000 × 106/L).

dMay

have neutrophil predominance early in infection, but lymphocyte predominance occurs after the first 6 to 48 hours. eThe

CSF-to-plasma glucose ratio is ≤0.40 in most patients.

fThe

likelihood of a positive Gram stain correlates with number of bacteria in the

CSF.

gThe

yield of positive results is significantly reduced by previous administration of antimicrobial therapy.

1


Central Nervous System Infections

CONT.

Meningitis can also result from medications; autoimmune diseases; malignancy; and other nonviral infectious causes, including spirochetes, fungi, or mycobacteria. Serology may be helpful to diagnose aseptic meningitis caused by syphilis, Borrelia burgdorferi (Lyme disease), or certain fungal organisms.

Treatment Anti-infective treatments are not available for viral meningitis with the exception of underlying herpes viruses. The primary focus of management in viral meningitis is to distinguish it from other causes of infectious meningitis, particularly bacterial meningitis. Symptomatic and supportive management for viral meningitis and rigorous evaluation to exclude bacterial meningitis are the cornerstones of management. Empiric antimicrobial agents may be initiated until full CSF profiles and cultures are finalized and bacterial meningitis can be excluded. Antibiotics can be started and continued as empiric treatment for 48 to 72 hours, after which most routine CSF cultures will show preliminary results. The increasing availability of enteroviral PCR testing can shorten the time required to exclude bacterial infection. Repeat lumbar puncture can be performed after the initial 2 to 3 days if lack of clinical response is a concern or if documenting the evolution of the CSF profile to that more characteristic of viral meningitis is desired.

Neisseria meningitidis) or travel to regions of the world where certain organisms are more prevalent and endemic (such as sub-Saharan Africa, where endemic meningococcal disease is present) will also increase the risk for bacterial meningitis. The two most common organisms causing bacterial meningitis are Streptococcus pneumoniae and N. meningitidis, which together are responsible for more than 80% of infections (Table 2).

Diagnosis Bacterial meningitis is classically described as producing fever, nuchal rigidity, and altered mental status. However, all three symptoms may not be present in many patients with confirmed disease. Other clinical manifestations that suggest bacterial meningitis include photophobia, headache (often a very severe headache distinctly different from what the patient may typically experience), and dermatologic manifestations (rash, petechiae, purpura) or neurologic abnormalities

Table 2.

Causes of Bacterial Meningitis

Infecting Organism

Cases (%)

Comments

Streptococcus pneumoniae

>70

Many cases caused by serotypes not covered by pneumococcal vaccination; functionally and anatomically asplenic patients at greater risk

Neisseria meningitidis

12

Can be transmitted through droplet exposure; functionally and anatomically asplenic patients at greater risk

Group B streptococci

7

Persons with diabetes, alcoholism, malignancy, liver disease at risk

Staphylococcus aureus

<5

Primary infection is associated with neurosurgery or CNS prosthetic device; secondary infection arises from bacteremia

Haemophilus influenzae

6

Decreased prevalence in the pediatric age group since type B conjugate vaccine; functionally and anatomically asplenic patients at greater risk

Listeria monocytogenes

<5

Immunosuppressed and decreased cell-mediated immunity (because of medications or medical conditions) as well as age (very old and very young) are risk factors

Gram-negative bacteria

<5

Nosocomial settings; complication of neurosurgery

Key Points

• Viral meningitis is most commonly caused by enteroviruses, herpes simplex virus type 2, and arboviruses. • Clinical signs and symptoms aiding in the diagnosis of viral meningitis include parotitis or orchitis with mumps virus, rash in enterovirus and HIV, pharyngitis with acute HIV, and genital lesions with herpes simplex virus.

HVC

• Viral meningitis is managed supportively, with empiric antimicrobials given only until cerebrospinal fluid cultures exclude bacterial meningitis.

bacterial Meningitis Causes Bacterial meningitis usually results from bacteremic dissemination of meningeal pathogens that colonize the nasopharynx or from another distant focus of infection but may also occur from contiguous spread of infection to the CSF (sinusitis or otitis media), direct inoculation (traumatic injury or neurosurgical procedure), or prosthetic device infection (CSF shunts or drains, intracranial pressure monitors). Individual host factors, such as colonization of the nasopharynx with potential meningeal pathogens, complement deficiency, anatomic or functional asplenia, glucocorticoid use, diabetes, hypogammaglobulinemia, and altered cellmediated immunity, can also contribute to increased susceptibility to acute bacterial meningitis. Exposure to other persons with bacterial meningitis (for certain pathogens, such as 2

CNS = central nervous system.


Central Nervous System Infections

CONT.

(focal neurologic or cranial nerve deficits, seizures). Physical examination maneuvers, such as testing for Brudzinski and Kernig signs, lack adequate sensitivity and specificity, although the presence of one or both of these findings is consistent with bacterial meningitis. Before proceeding with lumbar puncture, it should be determined whether CT of the head should be performed. CT of the head before lumbar puncture is indicated if signs or symptoms of increased intracranial pressure or a CNS mass lesion, such as papilledema, focal neurologic deficits, or altered mental status, are present; it is also indicated in patients who are immunocompromised or have a history of CNS disease (mass lesion, stroke, focal infection). In the absence of these findings, lumbar puncture can be undertaken without CNS imaging. Elevated CSF opening pressure is common. Characteristic CSF profiles in bacterial meningitis are outlined in Table 1. CSF Gram stain and cultures, when obtained before

antimicrobial therapy initiation, are usually diagnostic for the infecting organism. Rapid CSF tests, such as latex agglutination and molecular testing with PCR, can be supplemental diagnostic tools, although the diagnostic utility of latex agglutination testing is limited by a lack of specificity. Elevated CSF lactate levels suggest bacterial meningitis, but this is not a specific finding. Blood cultures, if obtained before antimicrobial therapy, are often positive and may be an adjunctive diagnostic tool in isolating the bacterial causative pathogen if CSF cannot be obtained before antibiotic administration.

Management Suspected bacterial meningitis demands prompt therapy; because of the risk for substantial morbidity and mortality with delayed treatment, empiric therapy is often initiated before CSF results are available (Figure 1). When a presumptive diagnosis of bacterial meningitis has been made, empiric

F i g u r e 1 . Management algorithm for adults suspected of having bacterial meningitis. CNS = central nervous system; c/w = consistent with; CSF = cerebrospinal fluid. Adapted with permission from Tunkel AR, Hartman BJ, Kaplan SL, et al. Practice guidelines for the management of bacterial meningitis. Clin Infect Dis. 2004 Nov 1;39(9):1270. [PMID: 15494903] Copyright 2004 Oxford University Press.

3


Each of the numbered items is followed by lettered answers. Select the ONE lettered answer that is BEST in each case.

Item 1 A 33-year-old woman is evaluated in the emergency department for a 2-month history of fever, lethargy, weight loss, and headache. She moved to the United States from India 4 years ago. Her father died of tuberculosis 20 years ago. Medical history is otherwise unremarkable, and she takes no medications. On physical examination, temperature is 38.6 °C (101.5 °F), blood pressure is 114/70 mm Hg, pulse rate is 94/min, and respiration rate is 18/min. BMI is 20. Except for lethargy, neurologic examination is unremarkable. No abnormalities are noted on ophthalmologic, cardiac, or pulmonary examinations or in the remainder of the physical examination. Cerebrospinal fluid (CSF) studies: Leukocyte count 275/µL (275 × 106), with 98% lymphocytes Glucose 30 mg/dL (1.7 mmol/L) Protein 250 mg/dL (2500 mg/L) CSF opening pressure 150 mm H2O The remainder of a complete blood count and comprehensive metabolic panel are normal. Acid-fast bacilli smear of CSF is negative, but polymerase chain reaction is positive for Mycobacterium tuberculosis. Minimal basilar meningeal enhancement is seen on CT scan of the head without any evidence of cisternal or ventricular abnormalities, midline shift, or mass lesion. In addition to four-drug antituberculous therapy, which of the following is the most appropriate additional treatment? (A) (B) (C) (D)

Acetazolamide Dexamethasone Furosemide Ventriculoperitoneal shunt

Item 2 A 33-year-old man is admitted to the hospital after experiencing a generalized tonic-clonic seizure. He has also had increasing weakness of the left hand and arm, headaches, and fever of 1 week’s duration. Medical history is significant for AIDS, without opportunistic infections in the past several years. His last clinic visit was more than 2 years ago, and his family reports that he has been taking his antiretroviral therapy (ART) only intermittently since around that time. On physical examination, temperature is 37.3 °C (99.1 °C), blood pressure is 142/92 mm Hg, pulse rate is 96/min, and respiration rate is 14/min. He is somnolent and slightly confused. His general medical examination is normal. On neurologic examination, there is no nuchal rigidity. Left upper extremity weakness is noted, but the examination is otherwise unremarkable.

Laboratory studies: CD4 cell count Leukocyte count

66/µL 4400/µL (4.4 × 109/L) (differential: 80% polymorphonuclear cells, 12% lymphocytes, 5% eosinophils, 3% monocytes) Negative

Cryptococcal antigen, serum Toxoplasma gondii IgG positive, IgM negative

Noncontrast head CT shows a single lesion in the right parietal area. Brain MRI with contrast confirms this lesion with additional smaller lesions in the left frontal, basal ganglia, and cerebellar areas; all show a small surrounding area of enhancement without edema. No meningeal enhancement is seen. Which of the following is the most likely diagnosis? (A) (B) (C) (D)

Central nervous system lymphoma Cryptococcosis Progressive multifocal leukoencephalopathy Toxoplasmosis

Item 3 A 59-year-old man is evaluated in the ICU for fever and leukocytosis. He was admitted to the ICU 13 days ago with respiratory failure resulting from Guillain-Barré syndrome and was intubated and mechanically ventilated. On hospital day 9, he developed fever without an increase in secretions or change in oxygenation. A new left lower-lobe infiltrate was seen on chest radiograph, and his leukocyte count was 17,500/µL (17.5 × 109/L). Sputum culture grew methicillin-sensitive Staphylococcus aureus. Nafcillin was started, with temporary resolution of the fever within 48 hours. Medical history is otherwise unremarkable. Medications are nafcillin and intravenous immune globulin. On physical examination, temperature is 38.2 °C (100.8 °F), blood pressure is 132/84 mm Hg, pulse rate is 94/ min, and respiration rate is 18/min. Pulmonary examination reveals decreased breath sounds in the left lower lung field. The remainder of the examination is noncontributory. Laboratory studies show a leukocyte count of 17,300/µL (17.3 × 109/L). Sputum Gram stain reveals 1+ leukocytes and 1+ gram-positive cocci in clusters. A new moderate left pleural effusion, but no increase in the left lower lobe infiltrate, is seen on chest radiograph. CT scan shows left lower lobe consolidation with air bronchograms and a moderate left pleural effusion. Which of the following is the most appropriate next step in management? (A) (B) (C) (D)

Add gram-negative antimicrobial coverage Change nafcillin to vancomycin Perform bronchoscopy with bronchoalveolar lavage Perform thoracentesis

107

Self-Assessment Test

Directions


Item 1

Answer:

B

Educational Objective: Treat a patient with tuberculous meningitis. This patient has tuberculous meningitis and should receive glucocorticoid therapy in addition to antituberculous therapy. Dexamethasone is recommended for adults with tuberculous meningitis because limited data show some mortality benefits. The recommended dose of dexamethasone is 12 mg/d for 3 weeks, with gradual tapering during the following 3 weeks. Some experts use prednisone instead of dexamethasone, and others recommend a slightly longer duration of glucocorticoid therapy (approximately 8 weeks). The recommendations for duration of treatment of extrapulmonary tuberculosis are generally the same as for pulmonary tuberculosis (6-9 months). However, the recommended treatment duration for tuberculous meningitis is longer (9-12 months), with the exact duration determined by response to therapy and antibiotic sensitivities of the isolate. Diuretics—such as acetazolamide and furosemide, which decrease cerebrospinal fluid (CSF) production by the choroid plexus—have been used as temporizing measures for management of hydrocephalus in patients who are not stable enough to undergo surgical decompression. However, there is no indication for their use in this patient. Patients with tuberculous meningitis can develop hydrocephalus, usually secondary to impaired CSF resorption. Ventriculoperitoneal shunts can be used to manage hydrocephalus. This patient has no evidence of hydrocephalus, which is typically associated with increased intracranial pressures and ventriculomegaly on brain imaging. Serial lumbar punctures after initiation of glucocorticoids and antituberculous medications can be attempted initially in patients with hydrocephalus before attempting surgical drainage with a shunt. Key PoinT

• Patients with tuberculous meningitis should receive a glucocorticoid in addition to antituberculous therapy.

Bibliography American Thoracic Society; CDC; Infectious Diseases Society of America. Treatment of tuberculosis [erratum in MMWR Recomm Rep. 2005 Jan 7;53(51):1203]. MMWR Recomm Rep. 2003 Jun 20;52(RR-11):1-77. [PMID: 12836625]

Item 2

Answer:

D

Educational Objective: Diagnose central nervous system toxoplasmosis. This patient most likely has central nervous system (CNS) Toxoplasma gondii infection. He has not been adherent to

antiretroviral therapy, and his CD4 cell count has decreased to less than 100/µL, putting him at risk for opportunistic infection. Clinical presentation of CNS toxoplasmosis typically involves new headaches and focal neurologic deficits and may also include fever and seizures. The findings on this patient’s MRI are classic for toxoplasmosis, with multiple ring-enhancing lesions. Because CNS toxoplasmosis in AIDS results from reactivation of earlier infection, patients usually have a positive result for IgG antibody but not IgM. He should be treated empirically and followed for clinical and radiographic response, which typically occurs within 1 to 2 weeks. Primary CNS lymphoma generally requires a more severe degree of immunosuppression than most other AIDS-related complications, most commonly occurring when the CD4 cell count drops below 50/µL; the incidence has decreased with effective antiretroviral therapy. Although primary CNS lymphoma may present with more than one lesion on MRI, it most often presents with a single large lesion. Primary CNS lymphoma is strongly associated with Epstein-Barr virus infection, and polymerase chain reaction of cerebrospinal fluid to detect Epstein-Barr virus can facilitate the diagnosis. The prognosis remains poor. Cryptococcal meningoencephalitis is another common CNS infection in AIDS but rarely results in focal mass lesions on imaging, more often presenting with mental status changes than focal deficits and seizure. The serum cryptococcal antigen and cerebrospinal fluid antigen are positive in 95% and 99% of cases, respectively; therefore, a negative antigen test result argues against this diagnosis. Progressive multifocal leukoencephalopathy presents in AIDS with focal deficits and mental status changes. Onset usually occurs over several weeks, compared with several days for toxoplasmosis, and fever is uncommon. Furthermore, progressive multifocal leukoencephalopathy lesions on MRI are typically white-matter, noninflammatory lesions (unless accompanied by immune reconstitution inflammatory syndrome) with minimal enhancement and no mass effect. Key PoinT

• Central nervous system toxoplasmosis in AIDS results from reactivation of earlier infection, typically involves new focal neurologic deficits and headaches, and may also include fever and seizures.

Bibliography Panel on Opportunistic Infections in HIV-Infected Adults and Adolescents. Guidelines for the prevention and treatment of opportunistic infections in HIV-infected adults and adolescents: recommendations from the Centers for Disease Control and Prevention, the National Institutes of Health, and the HIV Medicine Association of the Infectious Diseases Society of America. http://aidsinfo.nih.gov/contentfiles/lvguidelines/ adult_oi.pdf. Updated April 16, 2015. Accessed July 20, 2015.

135

Answers and Critiques

Answers and Critiques


Medical Knowledge Self-Assessment ProgramÂŽ

Nephrology

19 AMA PRA Category 1 Credits™ available until December 31, 2018.


Fluids and Electrolytes

Kidney Biopsy Kidney biopsy provides pathologic diagnostic information that can be useful for treatment, disease surveillance, and, potentially, prognosis. Kidney biopsy should be considered in patients with glomerular hematuria, severely increased albuminuria, acute or chronic kidney disease of unclear etiology, and kidney transplant dysfunction or monitoring. Some kidney diseases such as acute interstitial nephritis or atheroembolic disease may be occult and difficult to diagnose without biopsy. Direct visualization by ultrasound or CT is the standard for percutaneous kidney biopsy. Nonpercutaneous approaches for kidney biopsy include open, laparoscopic, or transjugular. Contraindications to kidney biopsy include bleeding diatheses, severe anemia (especially with patient refusal of blood transfusions), UTI, hydronephrosis, uncontrolled hypertension, anatomic abnormalities, renal tumor, atrophic kidneys, and an uncooperative patient. Solitary native kidney and pregnancy are not absolute contraindications, but careful consideration of the risks and benefits is necessary. Significant complications include pain, macroscopic hematuria, hemorrhage (potentially requiring transfusions, angiography with embolization, and surgery such as nephrectomy), loss of kidney function, and death. Key Point

• Kidney biopsy should be considered in patients with glomerular hematuria, severely increased albuminuria, acute or chronic kidney disease of unclear etiology, and kidney transplant dysfunction or monitoring.

Fluids and Electrolytes Osmolality and Tonicity Plasma osmolality is determined by the concentration of sodium and its accompanying anions, plasma glucose, and blood urea nitrogen (BUN). Total osmolality can be directly measured with an osmometer or may be calculated: Plasma Osmolality (mOsm/kg H2O) = 2 × Serum Sodium (mEq/L) + Plasma Glucose (mg/dL)/18 + BUN (mg/dL)/2.8 Although both methods indicate the overall osmolality of plasma, urea is freely diffusible across most cell membranes and does not exert a significant osmotic effect. Therefore, the effective osmolality (tonicity) is determined by subtracting the measured concentration of urea (divided by 2.8, if in mg/dL) from the measured osmolality. Under normal conditions, the effective osmolality is maintained in the range of 275 to 295 mOsm/kg H2O. Disorders of osmolality are usually caused by abnormalities in the relative ratio of sodium to body water. Two key effectors in maintaining normal plasma osmolality are thirst and the level of circulating antidiuretic hormone (ADH; also 10

known as arginine vasopressin), which promotes water reabsorption in the distal tubule and collecting duct of the kidney and has a peripheral vasoconstricting effect. Thirst and ADH release are stimulated by increases in plasma osmolality through central osmoreceptors in the hypothalamus. ADH release is also influenced by volume status; hypovolemia is associated with higher circulating ADH levels at each level of plasma osmolality.

Disorders of Serum Sodium Hyponatremia Hyponatremia, defined as a serum sodium concentration <136 mEq/L (136 mmol/L), most often results from an increase in circulating ADH in response to a true or sensed reduction in effective arterial blood volume with resulting fluid retention. Hyponatremia may also be caused by elevated ADH levels associated with the syndrome of inappropriate antidiuretic hormone secretion. Hyponatremia occurring with normal or suppressed ADH levels may be seen in the setting of very low solute intake (such as the “tea and toast” syndrome and “beer potomania”), in which inadequate amounts of solute are available to excrete an increased volume of ingested free water.

Evaluation The initial evaluation of patients with hyponatremia includes measurement of plasma and urine osmolality and urine sodium as well as a careful assessment of the volume status. Measurement of plasma osmolality is needed to exclude hyperosmolar hyponatremia and pseudohyponatremia (in which plasma osmolality is normal). Hyperosmolar hyponatremia results from elevated plasma glucose or from exogenously administered solutes such as mannitol or sucrose. Hyperglycemia causes the osmotic translocation of water from the intracellular to the extracellular fluid compartment, which results in a decrease in the serum sodium level by approximately 1.6 mEq/L (1.6 mmol/L) for every 100 mg/dL (5.6 mmol/L) increase in the plasma glucose above 100 mg/dL (5.6 mmol/L). Pseudohyponatremia is the result of a laboratory error in the measurement of serum sodium. Clinical laboratories measure the amount of sodium present in a dilute serum sample. The serum sodium concentration is then calculated based on the assumption that plasma is 93% water. However, in patients with extreme hyperlipidemia and/or hyperproteinemia, this assumption may not be correct because lipids or protein make up a greater percentage of overall plasma volume. This may result in the reporting of a falsely low serum sodium concentration. In patients with hypotonic hyponatremia, further evaluation is based on urine osmolality (Figure 5). Urine osmolality <100 mOsm/kg H2O is consistent with appropriately suppressed ADH release as seen in primary polydipsia or inadequate solute intake. Urine osmolality >100 mOsm/kg H2O indicates that ADH excess is playing a dominant role. Hypotonic


Fluids and Electrolytes

F i g u r e 5 . Evaluation of hypotonic hyponatremia. SIADH = syndrome of inappropriate antidiuretic hormone secretion; UNa = urine sodium concentration; Uosm = urine osmolality.

CONT.

hyponatremia with a urine osmolality >100 mOsm/kg H2O is classified into three groups based on the clinical volume status: hypovolemic, hypervolemic, and euvolemic.

Hypovolemic Hyponatremia Hypovolemia causes stimulation of the sympathetic nervous system, activation of the renin-angiotensin-aldosterone (RAA) axis, and release of ADH. These adaptive responses allow volume maintenance at the expense of a low serum sodium with excessive water intake. Extrarenal causes of hypovolemia are common and include loss of sodium from the gastrointestinal tract (vomiting or diarrhea) and insensible loss of sodium chloride (sweating, burns, respiratory tract); in these conditions, the urine sodium concentration is typically <20 mEq/L (20 mmol/L). Renal causes of hypovolemic hyponatremia result in excessive loss of salt and water in the urine with volume depletion; the urine sodium concentration is typically >20 mEq/L (20 mmol/L). Diuretic therapy is the most common cause. Less common are

adrenal insufficiency and salt-wasting nephropathies with impaired renal tubular function; typical causes include reflux nephropathy, interstitial nephropathies, post-obstructive uropathy, cystic kidney diseases, and the recovery phase of acute tubular necrosis. The syndrome of cerebral salt wasting is a rare cause due to inappropriate natriuresis from intracranial disease such as subarachnoid hemorrhage, traumatic brain injury, craniotomy, encephalitis, and meningitis.

Hypervolemic Hyponatremia Patients with hypervolemic hyponatremia have increased total body sodium and water, with the latter dominating and leading to a reduced serum sodium concentration. The pathophysiology of hyponatremia in sodium-avid edematous disorders (heart failure, cirrhosis, and the nephrotic syndrome) is similar to that in hypovolemic hyponatremia, with the kidney sensing a decreased arterial blood volume despite excess total body sodium and water. The urine sodium concentration is typically <20 mEq/L (20 mmol/L) in the absence of diuretic therapy. 11


Fluids and Electrolytes

CONT.

However, in patients with hypervolemic hyponatremia due to acute or chronic kidney failure, there is an inability to effectively manage both sodium and water, leading to an excess of water relative to sodium. In acute or chronic kidney failure, the urine sodium concentration is typically >20 mEq/L (20 mmol/L).

Euvolemic Hyponatremia Euvolemic hyponatremia results from abnormalities in maintaining sodium and water balance in the setting of normal total volume or to a reset osmostat. Because the effective arterial blood volume is normal, the urine sodium concentration is usually >20 mEq/L (20 mmol/L). The most common cause of euvolemic hyponatremia is the syndrome of inappropriate antidiuretic hormone secretion (SIADH) (Table 6). It is important to exclude hypothyroidism and glucocorticoid insufficiency in patients with euvolemic hyponatremia and suspected SIADH because thyroid hormone and endogenous glucocorticoid are necessary for normal free water excretion by the kidney. Reset osmostat refers to a downward setting of the level at which sensors of plasma osmolality trigger the release of antidiuretic hormone and is associated with quadriplegia, tuberculosis, advanced age, pregnancy, psychiatric disorders, and chronic malnutrition. This lowered setpoint leads to mild hypo-osmolar euvolemic hyponatremia.

Clinical Presentation Patients with acute symptomatic hyponatremia (occurring in <48 hours) often present with symptoms and signs ranging from headache, nausea, and/or vomiting, to altered mental

TABlE 6. Causes of the Syndrome of Inappropriate Antidiuretic Hormone Secretion Central nervous system disorders: Guillain-Barré syndrome; hemorrhagea; infections; inflammatory disorders; mass lesionsa; multiple sclerosis Drugs: 3,4-methylenedioxymethamphetamine (also known as ecstasy)a; antipsychotic medications; carbamazepine; chlorpropamide; clofibrate; cyclooxygenase-2 inhibitorsa; cyclophosphamide; desmopressina; ifosfamide; nicotine; NSAIDsa; opiatesa; phenothiazines; selective serotonin reuptake inhibitorsa; serotonin-norepinephrine reuptake inhibitorsa; tricyclic antidepressants; vasopressina; vincristine Endurance exercise (marathon running) Familial disorders Infections: HIV infection; Rocky Mountain spotted fever Postoperative setting: anesthesiaa; nauseaa; paina Pulmonary disorders: infectionsa; inflammatory disorders; positive pressure mechanical ventilationa; respiratory failure Tumors: gastrointestinal tract tumors; genitourinary tract tumors; lymphomas; respiratory tract tumorsa; sarcomas; small cell cancera; thymomas aMost

12

common causes.

status, seizures, obtundation, central herniation, or death. Acute symptomatic hyponatremia can occur in several clinical settings and is a medical emergency because a sudden drop in serum sodium concentration can overwhelm the capacity of the brain to regulate cell volume, leading to massive cerebral edema. Due to cerebral adaptation, patients with chronic hyponatremia (developing over >48 hours) are less likely to exhibit severe symptoms and may be completely asymptomatic. Key Points

• Initial evaluation of hyponatremia includes measurement of plasma and urine osmolality and urine sodium as well as clinical assessment of volume status. • Hypotonic hyponatremia evaluation begins with urine osmolality; urine osmolality <100 mOsm/kg H2O suggests appropriately suppressed antidiuretic hormone (ADH), whereas urine osmolality >100 mOsm/kg H2O suggests ADH excess.

Management In patients with hyponatremia, the severity of symptoms determines the urgency and intensity of therapy. Additionally, when treating hyponatremia, the rate of correction of serum sodium concentration must be carefully considered to avoid the osmotic demyelination syndrome (ODS). Brain cells adapt to chronic hyponatremia by reducing intracellular concentration of organic osmolytes to cope with hypotonicity. Because of this, rapid correction (>8.0 mEq/L [8.0 mmol/L] within the first 24 hours and/or >16 mEq/L [16 mmol/L] within the first 48 hours) may cause neuronal cell damage with resulting demyelination. ODS classically affects the pons, resulting in central pontine myelinolysis; patients may present days after overcorrection of hyponatremia with para- or quadriparesis, dysphagia, dysarthria, diplopia, a “locked-in” syndrome, and/ or loss of consciousness.

Acute Symptomatic Hyponatremia Initial treatment of acute symptomatic hyponatremia includes hypertonic saline to acutely increase serum sodium concentration by 1.0 to 2.0 mEq/L (1.0-2.0 mmol/L) per hour to a total increase of 4.0 to 6.0 mEq/L (4.0-6.0 mmol/L) within the first 6 hours; this increase is typically sufficient to alleviate acute symptoms, avoiding overly rapid correction as previously discussed. The achieved serum sodium should be maintained at a constant level for the first 24 hours. Because the response to hypertonic saline can be highly unpredictable due to a rapid reduction in circulating ADH and the excretion of a dilute urine, frequent (every 1 to 2 hours) measurements of serum sodium concentration are necessary. Should the serum sodium concentration be inadvertently overcorrected, hyponatremia can be safely re-established by administration of the vasopressin agonist desmopressin and intravenous 5% dextrose in water, with monitoring of the serum sodium concentration.


Fluids and Electrolytes

CONT.

Intravenous loop diuretics increase free water excretion by interfering with the renal countercurrent multiplication system and may facilitate acute treatment. Vasopressin antagonists do not have a role in the management of acute symptomatic hyponatremia.

Chronic Hyponatremia In patients with hypovolemic hyponatremia, diagnosis and correction of the underlying cause of hypovolemia are indicated. In patients requiring specific treatment, volume repletion with oral salt supplementation or intravenous isotonic saline suppresses circulating ADH and induces a water diuresis. Management of hypervolemic hyponatremia includes treatment of the underlying disorder, sodium and free water restriction (in practical terms, fluid restriction), and diuretic therapy. In patients with heart failure, vasopressin antagonists (such as tolvaptan and conivaptan) are effective in normalizing hyponatremia; however, use of these agents should be limited to patients with a serum sodium concentration <120 mEq/L (120 mmol/L) despite optimal therapy with diuresis and RAA system inhibition. Neither tolvaptan nor conivaptan is currently recommended for the treatment of hyponatremia associated with cirrhosis. The initial treatment of euvolemic hyponatremia includes management of the underlying cause if possible. Fluid restriction without limiting sodium intake is a cornerstone of therapy. Patients who do not respond to fluid restriction can be treated with loop diuretics combined with oral salt supplementation. Oral demeclocycline, which induces nephrogenic water loss, can be utilized to treat SIADH that fails fluid restriction or furosemide plus salt. However, demeclocycline can be associated with acute kidney injury from natriuresis and/or direct renal toxicity and is contraindicated in patients with cirrhosis. Vasopressin antagonists (oral tolvaptan and intravenous conivaptan) have been shown to be effective in normalizing serum sodium concentration in SIADH. Oral tolvaptan should be reserved for the management of serum sodium concentrations <120 mEq/L (120 mmol/L) and persistent SIADH that has failed fluid restriction and/or oral furosemide and salt tablets. Liver chemistry test abnormalities have been reported with chronic tolvaptan therapy; therefore, the use of this drug should be limited to less than 1 to 2 months, with close laboratory monitoring. Key Points

• Initial treatment of acute symptomatic hyponatremia includes hypertonic saline to acutely increase serum sodium concentration by 1.0 to 2.0 mEq/L (1.02.0 mmol/L) per hour to a total increase of 4.0 to 6.0 mEq/L (4.0-6.0 mmol/L) within the first 6 hours; the achieved serum sodium should be maintained for the first 24 hours. (Continued)

K e y P o i n t s (continued)

• Chronic asymptomatic hypervolemic hyponatremia management includes treatment of the underlying disorder, sodium and free water restriction, and diuretic therapy. • First-line therapy of euvolemic hyponatremia includes treatment of the underlying cause and free water restriction, and second-line therapy includes loop diuretics combined with oral salt supplementation followed by oral demeclocycline; vasopressin antagonists should be reserved for patients with a serum sodium concentration <120 mEq/L (120 mmol/L) who are unresponsive to first- and second-line therapy.

Hypernatremia Hypernatremia, defined as a serum sodium concentration >145 mEq/L (145 mmol/L), is often the result of a combined water and volume deficit, with losses of water in excess of sodium. Considerably less common than hyponatremia, hypernatremia is associated with mortality rates of approximately 40% to 60%. Elderly individuals with reduced thirst and/or diminished access to fluids are at the highest risk of developing hypernatremia. Patients with hypernatremia may rarely have a central defect in hypothalamic osmoreceptor function, with both decreased thirst and reduced ADH secretion. More commonly, hypernatremia may develop following the loss of free water via renal or extrarenal routes. Insensible losses of water due to evaporation from the skin or respiratory tract may increase in the setting of fever, exercise, heat exposure, severe burns, or mechanical ventilation. Osmotic diarrhea and viral gastroenteritides typically generate hypo-osmotic stools, leading to significant free water loss and hypernatremia. Secretory diarrheas typically do not result in hypernatremia; rather, the isotonic stool loss causes hypovolemia with or without hypovolemic hyponatremia. Common causes of renal water loss include osmotic diuresis secondary to hyperglycemia, post-obstructive diuresis, or drugs (contrast, mannitol). Water diuresis occurs in central or nephrogenic diabetes insipidus. Central diabetes insipidus results from inadequate secretion of ADH from the hypothalamus (see MKSAP 17 Endocrinology and Metabolism, Disorders of the Pituitary Gland). Nephrogenic diabetes insipidus results from resistance of the kidney to the action of ADH and has multiple causes. Gestational diabetes insipidus is a rare complication of pregnancy wherein increased activity of a placental protease with “vasopressinase” activity leads to reduced circulating ADH. See Table 7 for more information.

Evaluation In patients with hypernatremia, the history should focus on the presence or absence of thirst, polyuria, and/or an extrarenal source for water loss such as diarrhea. The physical examination should include a neurologic examination and assessment of volume status; accurate documentation of daily 13

HVC


Medical Knowledge Self-Assessment ProgramÂŽ

Pulmonary and Critical Care Medicine

19 AMA PRA Category 1 Credits™ available until December 31, 2018.


Each of the numbered items is followed by lettered answers. Select the ONE lettered answer that is BEST in each case.

Item 1 A 55-year-old man is evaluated in the emergency department after being rescued from his burning home by firefighters. His medical history is notable for COPD, and his only medication is an albuterol-ipratropium metered-dose inhaler. On physical examination, he is alert and in pain. Temperature is 37.8 °C (100.0 °F), blood pressure is 124/60 mm Hg, pulse rate is 116/min, and respiration rate is 22/min; BMI is 31. Oxygen saturation is 98% breathing 3 L/min oxygen by nasal cannula. Soot is noted in the nares and throughout the oral pharynx. The oral mucosa is edematous. Burns are noted on the right upper extremity and lower chest. Cardiac examination reveals a regular rhythm. Pulmonary examination reveals a monophonic wheeze over the anterior chest, diffuse expiratory wheezes, and increased work of breathing. Arterial blood gas studies breathing 30% oxygen reveal a pH of 7.32, a Pco2 of 50 mm Hg (6.7 kPa), and a Po2 of 78 mm Hg (10.4 kPa). A chest radiograph shows increased lung volumes consistent with hyperinflation but no infiltrates. Which of the following is the most appropriate next step in treating this patient’s respiratory findings? (A) (B) (C) (D) (E)

Which of the following is most likely to prevent deconditioning in the ICU? (A) Passive range of motion exercises only while in the ICU (B) Progressive physical activity as tolerated starting now (C) Progressive physical activity as tolerated following discontinuation of vasopressors (D) Progressive physical activity as tolerated following extubation

Item 3 A 58-year-old woman is evaluated for a right pulmonary nodule that was discovered incidentally 3 weeks ago. She is currently asymptomatic and has not had shortness of breath, fever, chills, weight loss, or night sweats. Medical history is otherwise unremarkable, and she takes no medications. She is a life-long nonsmoker. On physical examination, temperature is 37.1 °C (98.8 °F), blood pressure is 126/82 mm Hg, pulse rate is 68/min, and respiration rate is 10/min; BMI is 30. There is no cervical or supraclavicular lymphadenopathy. The lungs are clear to auscultation. No clubbing is noted. The 5-mm nodule seen on CT is shown.

Administer a helium-oxygen mixture Administer methylprednisolone Perform endotracheal intubation Start nebulized epinephrine Start noninvasive ventilation

Item 2 A 72-year-old man is admitted to the ICU for severe community-acquired pneumonia. He was admitted to the hospital 2 days ago with cough and dyspnea. Despite appropriate intravenous antibiotics, his respiratory status declined and he was transferred to the ICU, was intubated, and was placed on mechanical ventilation. A vasopressor was needed for persistent hypotension. Since ICU admission 36 hours ago, he has had stable oxygenation and blood pressure and his oxygen and vasopressor dose are being decreased. Medical history is otherwise unremarkable. Medications are ceftriaxone, azithromycin, norepinephrine, and low-molecular-weight heparin prophylaxis. On physical examination, the patient is intubated but responsive. Temperature is 38.3 °C (100.9 °F), blood pressure is 95/58 mm Hg, and pulse rate is 110/min; BMI is 27. Chest examination shows decreased breath sounds at the left lung base. Cardiac examination reveals a grade 2/6 systolic flow murmur. The remainder of the examination is unremarkable. Laboratory studies are significant for a leukocyte count of 15,000/µL (15 × 109/L); the complete blood count is otherwise normal. Complete metabolic profile is normal. Blood and sputum cultures are negative since admission. A chest radiograph is significant for left lower lobe consolidation but is otherwise unremarkable.

Which of the following is the most appropriate next step in management? (A) (B) (C) (D)

Bronchoalveolar lavage PET/CT scan Review any previous chest imaging Transthoracic needle biopsy

Item 4 A 62-year-old man is evaluated for declining exercise capacity over the past year. He was diagnosed with moderate COPD 3 years ago. His symptoms had previously been well controlled with tiotropium and as-needed albuterol. He has 95

Self-Assessment Test

Directions


Answers and Critiques Answer:

C

Educational Objective: Treat thermal upper airway injury with endotracheal intubation. The most appropriate next step in treating this patient’s respiratory findings is to perform endotracheal intubation. This patient sustained significant thermal injury to his upper airway as evidenced by an edematous oropharynx and monophonic wheeze consistent with inspiratory stridor. Intense heat can cause edema and blistering from the mouth to the larynx, and patients with a visibly damaged airway or stridor are at high risk of complete upper airway obstruction due to swelling. It is difficult to predict which patients with acute inhalational injury will develop complete upper airway obstruction, and even patients with minimally apparent inhalational airway injury may develop worsening obstruction associated with intravenous hydration or analgesia needed for burn treatment. Therefore, ensuring upper airway patency is the first priority in patients with significant smoke exposure, and endotracheal intubation is indicated in this patient at high risk of obstruction. Because helium is less dense than nitrogen, a mixture of helium and oxygen (heliox) may be used in situations in which there is increased airway resistance. The combination of helium and oxygen improves laminar airflow and may decrease the work of breathing in patients with increased airway resistance. However, it would not treat the underlying inhalation-associated airway obstruction. Methylprednisolone is useful for COPD exacerbations, but systemic glucocorticoids do not have a role in treating upper airway edema or smoke-induced injury of the lower airways. Nebulized epinephrine can transiently reduce upper airway edema from allergic or anaphylactic reactions, but it does not have a role in the treatment of inhalational injury. This patient’s expiratory wheezes and acute respiratory acidosis suggest the presence of a COPD exacerbation triggered by smoke inhalation, in which case noninvasive ventilation would be appropriate. However, noninvasive ventilation will not prevent this patient from developing complete airway obstruction, and securing the airway with an endotracheal tube while it is still possible takes precedence. Key PoinT

• In patients with significant smoke inhalation, a visibly damaged airway and stridor are indicators of a high risk for complete upper airway obstruction due to swelling; such patients require immediate endotracheal intubation.

Bibliography Rex S. Burn injuries. Curr Opin Crit Care. 2012 Dec;18(6):671-6. [PMID: 23037877]

Item 2

Answer:

B

Educational Objective: Perform early mobilization in a critically ill patient. The most appropriate management is early mobilization with progressive physical activity as tolerated starting now. Disabling weakness and motor function impairments are common in ICU survivors and can last up to 5 years following critical illness. Traditionally, patients in the ICU were believed to be too ill to participate in significant physical activity, and any attempt at full mobilization and exercise was reserved until more aggressive therapies were withdrawn or the acute illness had resolved. This was especially true of patients on mechanical ventilation and vasopressors. In many cases, only passive range of motion exercises were provided in the ICU until the patient was believed to be appropriate for mobilization and increased amounts of active physical activity. However, studies have shown that exercise and physical therapy are the only interventions that have a significant effect on decreasing weakness and motor function deficits associated with critical illness. Additionally, it has been demonstrated that patients on ventilators and low-level vasopressors may safely participate in a progressive mobility program once stability is established. Early mobilization is defined as patient mobilization (such as sitting up in bed and dangling feet over the edge of the bed) within 48 hours of admission to the ICU. Once the patient is awake and interactive, more aggressive bed exercises can be done. If these activities are tolerated without instability, the patient should be assessed for the next level of activity. Early mobilization, along with careful attention to management of pain, agitation, and delirium, has been shown to reduce ICU and hospital length of stay, shorten return to independent functional status, and improve survival. Waiting for discontinuation of vasopressors or extubation is not appropriate because these may not occur for a significant period of time, in which substantial deconditioning may develop. Key PoinT

• Early mobilization with progressive physical activity, along with careful attention to management of pain, agitation, and delirium, has been shown to reduce ICU and hospital length of stay, shorten return to independent functional status, and improve survival.

123

Answers and Critiques

Item 1


Turn static files into dynamic content formats.

Create a flipbook
Issuu converts static files into: digital portfolios, online yearbooks, online catalogs, digital photo albums and more. Sign up and create your flipbook.